Foundation Course: Physics - Std. Ix

You might also like

Download as pdf or txt
Download as pdf or txt
You are on page 1of 47

FOUNDATION COURSE

F O R J E E (MAIN & ADV.)| NEET| K V P Y | N T S E | O LY M P I A D S |

PHYSICS |STD. IX
MOTION

Corporate Office : PARUS LOK COMMERCIAL COMPLEX


Boring Road Crossing, Patna-1 ; NEAR QUALITY CORNER
Helpline No. : 9569668800|Call-7544015993/4/6/7
CBSE | MOTION [3]

Chapter-1

Motion
1.1 INTRODUCTION 1.2 REVIEW OF BASIC CONCEPTS
The physics of motion focuses on the language, Particle
principles and laws, which describe and explain the
An object of negligible dimensions, i.e., a point mass is
motion of objects. The efforts should concentrate
known as a particle. This is only mathematical idealization.
around the m eaning of the inf orm ation and its
Even earth can be treated as a particle if we consider the
applications. motion of earth around the sun. The radius of the earth is
Mechanics is the branch of physics that focuses on the very small when compared with the distance between the
motion of objects and the forces that cause the motion earth and the sun. Similarly, a bus travelling a distance of
100 km can be considered as a particle, as the size of the
to change. There are two branches of Mechanics:
bus is very small compared to the distance travelled by it.
Kinematics and Dynamics. The word kinematics comes
from the Greek word ‘Kinema’ meaning motion. The Reference point
word dynamics comes from the Greek word ‘dynamis’
meaning power. Kinematics deals with the concepts that A fixed point or a fixed object with respect to which the
are needed to describe motion without any reference given body changes its position is known as reference
point. For example, a pillar can be a reference point for
to forces or the cause of the motion. The goal of study
an object crossing it.
of kinematics is to develop sophisticated mental models
which serve us in describing the motion of real world Rest and Motion
objects.
Rest : A body is said to be at rest if it does not change its
The language of kinematics deals with the words position with respect to the reference point. The objects
Vectors, Scalars, Distance, Displacement, Speed, which remain stationary at a place and do not change
Velocity, Acceleration, which are used with regularity their position are said to be at rest.
to describe the motion of objects. Example : A table and the chairs lying in a room are in a
Dynamics deals with the relation between the forces state of rest, because they do not change their position
and motion. The underlying concepts and principles of with respect to the surroundings of the room. Similarly,
buildings, pillars, monuments like The Taj Mahal are at
Physics have a mathematical basis. Throughout the
rest with respect to their surroundings.
course of study of physics; the emphasis will often be
upon the conceptual nature of physics, however we will Motion : An object is said to be in motion if it changes its
give considerable and persistent attention to its position from one place to another. In other words, the
mathematical aspect. movement of an object is known as the motion of the
object.
The motion of object can be described by words such
Examples : Cars, cycles, motorcycles, scooters, buses,
as distance, displacem ent, speed, velocity and
rickshaws, trucks etc. running on the road. Birds flying in
acceleration. Before we witness the concepts of
the sky. A lion chasing its prey. Fishes swimming in water,
kinematics, let’s review some basic concepts.
are examples of objects in motion.

CORPORATE OFFICE : Parus Lok Complex, Boring Road Crossing, Patna - 1


[4] PHYSICS | STD.IX

Rest and Motion are relative mathematical equations. In this chapter, mathematical
equations as well as graphical methods are used to
While sitting in a moving train, your distances from the describe the motion of a particle.
walls, roof and the floor of the compartment do not change.
That is, with respect to the compartment, your position 1.3 DISTANCE TRAVELLED AND
does not change. You are at rest with respect to the DISPLACEMENT
compartment. But your distance from the platform, from
which you boarded the train, changes as time passes. So When a body is displaced from one point to another, it
you are apparently moving with respect to the platform. describes a certain curve, which is called a trajectory of a
This means that an object can be at rest with respect to moving body. We call the motion of a particle as rectilinear
one thing and in motion with respect to some other thing or straight line if its trajectory is a straight line.
at the same time. So motion is not absolute; it is relative.
The length of the trajectory along which a body moves for
a certain time interval is said to be the path or distance
traversed by the body during that time interval. The
distance travelled is a physical quantity. It is measured
with a special unit of length, metre (m).
The actual length covered by a moving body in
between two points, irrespective of the direction in
which the body is moving, is called distance.
In the figure shown, if the particle performs one complete
rotation, then the distance covered by it is 2r .

When you are moving in a train, you are both


at rest and motion relatively
The straight line distance between the initial and the final
Is the platform at rest, or is it moving with respect to the positions of a body is called the magnitude of the
compartment? displacement. The direction of the displacement is given
The distance of the platform from the compartment is by the direction from the initial position to the final position.
changing as time passes. So the platform is moving with The shortest distance covered by a moving body in
respect to the compartment. Also, the compartment is between two points, in a particular direction, is called
moving with respect to the platform. the displacement.
Suppose two trains are moving on parallel tracks in the The displacement of a particle and the distance travelled
same direction. Both started together and are moving at by it are two different quantities. Distance travelled in a
the same speed. The distance of a person A sitting in the given time has just a magnitude (numerical value) and
first train from another person B sitting in the other train no direction, whereas the displacement has magnitude
will not change. So A is at rest with respect to B. Similarly, as well as direction. It is important to note that the
B is at rest with respect to A. Both are moving with respect magnitude of the displacement is not always the same as
to the platform, but they are at rest with respect to each the distance travelled. If a particle moves in a plane along
other. a zig-zag path, the distance travelled in a given time
interval may be much greater than the magnitude of its
We are going to deal with the rectilinear motion of an displacement in the same time interval.
object in this chapter. A broad range of physical situations
For example : Consider a particle which moves from A
are examples of straight line or rectilinear motion:- A falling
to B along a curve in the figure. The distance travelled is
stone; an accelerating train; A braking car; A sliding hockey
equal to the length of the curve, whereas the magnitude
puck; A crate being pulled up a ramp, etc. We describe
of the displacement is equal to the length of the straight
the motion of a particle in two ways: Mathematical equation
line AB.
and Graphical method. The mathematical approach is
usually better for solving problems, because it permits A B
more precision than graphical methods. The graphical
method often provides more physical insight than a set of
A B

CORPORATE OFFICE : Parus Lok Complex, Boring Road Crossing, Patna - 1


CBSE | MOTION [5]

Even if the particle moves along a straight line, the Illustration 1 : A body is moving along a circular
distance travelled may be greater than the magnitude of path of radius R. What will be distance and displace-
its displacement. For example, if a particle goes from A to ment of the body when it completes half a revolution?
B on a straight line and returns to A, the displacement is
zero but the distance traversed is not zero. Only if the Sol. Distance = length of arc = R
particle moves along a straight line without changing its A B
direction, the magnitude of its displacement is equal to Magnitude of Displacement = straight line distance
the distance traversed as shown in above figure. between points A and B = 2R
(which is diameter of the circle)
For example : Consider the following motion depicted in
the figure. A man walks 4 metres east, 2 metres south, 4 Illustration 2 : A body moves 4m towards east and
metres west and finally, 2 metres north. then 3m north. What are the magnitude of displacement
4m and distance covered by the body ?
N
Sol. Distance = AB + BC = (4 + 3)m = 7m
2m 2m W E Magnitude of Displacement

4m S  AC  AB2  BC2  16  9  5m
Though the man walked a total distance of 12 metres, his C
displacement is zero metres. During the course of this N
motion, he has covered 12 metres of ground (distance =
W E 3m
12m).
S A B
Yet when he has finished walking, he is not “out of place” 4m
i.e., there is no displacement for his motion (displacement
= 0 m). Displacement, being a vector quantity, must give Illustration 3 : An athlete completes one quarter part
attention and regard to direction. The 4 metres east is of a circular track of radius R/2. Calculate the
cancelled by the 4 metres west; and the 2 metres south magnitude of displacement and distance covered by the
is cancelled by the 2 metres north. athlete.

Both distance and displacement are measured in same Sol. Magnitude of displacement is length of line AB i.e.
units. In SI and CGS systems, the units are metre and line joining initial postion to final position. From triangle
centimetre, respectively. OAB A
C
Difference between Distance and R2 R2 R
AB    R/2
Displacement 4 4 2
B O
R/2
Dist anc e D is plac em e nt
It is d efin ed as the actu al It is th e sh or te st dista nce R
pa th tra ver sed b y a b etwe en two p oin ts in a and distance is length of path ACB 
bo d y. p ar ti cula r d ire ction . 4

1.4 SPEED
It is a sca la r qu a ntity. It is a vecto r qu a ntity.
It ca n n eve r be ne ga tive It can b e n eg ati ve, zer o or
or ze ro for a mo vi ng
p ositi ve for a mo ving bo dy. Have you ever wondered; how the rate of motion of a
bo d y.
Nu me rica lly, dista nce body is measured? This can be understood from two
Nu me ri call y, di spla ce me nt of
tra ve ll ed b y a b od y can terms i.e., speed and velocity. Let us first discuss about
a bo dy ca n be eq ua l to o r
be eq ua l to o r gr ea ter
tha n d ispl ace me nt.
le ss tha n d istan ce tra vel led . speed.
Dista nce tr ave lle d is n ot When we consider speed, we consider the total distance
Di spla ce me nt is a un iq ue
a u niq u e p ath b etwe en
two p oin ts.
p ath b etw ee n two p o ints. covered with respect to time.
Th e di sta nce be twe en Assume that a rabbit is resting at A and in search of food
Di spla ce me nt be twee n two
two p oin ts give s
com pl ete in for ma ti on o f
p oi nts do es n ot gi ve com pl ete it moved along path ABCDEF and reached F.
in for ma ti on o f th e typ e of p ath
the typ e o f p ath fo llo we d 40 m
b y the bo d y. fo llo wed by the bo d y. C D

Dista nce ne ver


Di spla ce me nt can d ec rea se 20 m 20 m
de cre ase s with tim e. For 60 m
wi th time . Fo r a m ovin g b od y
a m ovin g b od y, i t is
it can be zer o. A 40 m B E F
ne ver zer o.

CORPORATE OFFICE : Parus Lok Complex, Boring Road Crossing, Patna - 1


[6] PHYSICS | STD.IX

If the rabbit takes 2 minutes to reach F from A, then we 6 a.m. 7 a.m. 8 a.m. 9 a.m.
say that speed of the rabbit is 1.5 m/s
Let us see how we get this 40 km 20 km 30 km
Total distance covered = 40 + 20 + 40 + 20 + 60 = 180 m 3 hours

Total time taken = 2 minutes = 120 s What do you observe?


We observe that the car covers 40 km from 6 a.m. to 7
180
 Speed  120  1.5m / s a.m., 20 km from 7 a.m. to 8 a.m. and 30 km from 8 a.m.
to 9 a.m. Here, the distance covered by the body is not
Thus, speed can be defined as the distance covered by a same in equal intervals of time, or equal distances in
body in unit time and is represented with v. unequal intervals of time. In such cases, we say that the
body is moving with non-uniform speed or variable speed.
distance covered s
Speed = v When a body covers unequal distances in equal intervals
time t
of time or equal distances in unequal interval of time then
Units : In CGS system, the unit of speed is centimetre the body is said to be moving with a non-uniform speed.
per second (cm/s) whereas in SI system, it is metre per
second (m/s). Examples : A train starting from a station, a dog chasing
a cat, have variable speeds.
Note:
(i) Speed is a scalar quantity as it has only magnitude
Average Speed
without any specific direction. Let us consider the motion of the car in the previous case.
(ii) If the distances covered are very small, then the The car is covering unequal distances in equal intervals
speed is expressed in cm/s or cms–1. of time.

(iii) If the distances covered are fairly large, the speed How do you calculate the speed of the car for the whole
is expressed in km/h or kmh–1. journey?
In such cases, we consider the average speed of the car.
5
(iv) 1km per hour  metre per second Average speed of a body is the total distance covered by
18 it in total time.

Uniform Speed  Average speed = Total distance covered


Total time taken to cover the distance
Observe the motion of the car shown below:
In the previous case, the total distance covered is (40 +
4 p.m. 5 p.m. 6 p.m. 20 + 30) km = 90 km, whereas the total time taken is 3
hours.

30 km 30 km 90km
 Average speed  3h
 30km / h.
What do you observe?
The car covers 30 km in every hour. Then we say that Illustration 1 : A car travels 30 km at a uniform speed
the car is moving with a uniform speed. Hence, of 40 km/h and the next 30 km at a uniform speed of
When a body covers equal distances in equal intervals of 20 km/h. Find its average speed.
time (however small the time intervals may be), the car is
distance
said to be moving with a uniform speed. Sol. : We know that, speed =
time
Examples : A rotating fan, a rocket moving in space,
So, let the time taken to cover first 30 km is t1
etc., have uniform speeds. Similarly, an aeroplane flying
at 200 km/h or a train running at 90 km/h are moving with 30 3
uniform speeds. t1  hours  hours
40 4
Non-Uniform Speed and let the time taken to cover next 30 km is t2

Let us understand the same by observing the motion of 30 3


the following car. t2  hours  hours
40 2

CORPORATE OFFICE : Parus Lok Complex, Boring Road Crossing, Patna - 1


CBSE | MOTION [7]

3 3 9 given interval of time, one needs to know the direction in


So, the total time    hours which the body has moved. Suppose, I tell you that at
4 2 4 10.30 a.m. a ball was placed at the centre of a field. A boy
Total distance 60 60  4 pushed the ball along the ground to move it with a uniform
Average speed =   speed of 0.5 m/s.
Total time 9 9
4 Can you find the location of the ball after 1 minute ?
 Average speed = 26.6 km/hr You can only calculate the distance covered by the ball in
1 minute.
Illustration 2 : On a 120 km track, a train travels the
first 30 km with a speed of 30 km/hr. How fast must the S = vt = (0.5 m/s) (60 s) = 30 m
train travel the next 90 km so as to get an average speed The ball is 30 m away from the centre of the field. But this
of 60 km/hr for the entire trip ? does not tell us where exactly the ball is, i.e., 30 m from
Sol. : Let the speed of the train be u km/h for next 90 the centre in which direction? To locate the position of the
km. Then, ball, you need to know the direction in which the ball moved
with a speed of 0.5 m/s. If we say that the boy pushed the
90 ball towards the north, it means the ball moved with a
t2  , where t2 is time taken to cover 90 km speed of 0.5 m/s towards north. You can then locate the
u
ball at a point 30 m north from the centre of the field.
and time taken to cover first 30 km is say t1.
When speed and direction are both specified, we get the
Then, t 
30
1  1hr. velocity of the object. In the above example, we say that
30 the velocity of the ball is 0.5 m/s, north. This is equivalent
90 to the statement that the speed of the ball is
So, total time  t1  t 2  1 0.5 m/s, and it is moving towards the north.
u
The velocity of an object is a quantity that gives the speed
Total distance of the object as well as its direction of motion.
Average speed =
Total time Velocity has magnitude as well as direction. It is a vector
quantity.
120
 60   u  90 km / hr We can also define velocity as follows:
90
1 The velocity of an object is the displacement of the
u
object in a short time interval divided by the time
Illustration 3 : A train 150 m long is moving with a interval or the velocity of an object is its displacement
speed of 60 km/hr. In what time shall it cross a bridge per unit time.
1 km long ? The unit of velocity is the same as the that of speed. Thus
Sol. : Length of the bridge = 1 km = 1000 m the SI unit of velocity is metre per second, written as m/s.
We often use the unit km/h for convenience.
distance travelled by train = 1000 m + 150 m
= 1150 m. Velocity of an object moving along a straight Line
When an object moves along a straight line, there are
5
speed = 60 km/hr. = 60  m/s only two possible directions of motion. In such a case, its
18 velocity may be represented in a very simple manner. We
distance write the speed of the object, and put a plus sign before
 time taken =
speed
it if the object is moving in the positive direction of the
line, and a minus sign if it is moving in the negative
1150 m direction of the line. The resulting number gives the speed
t= m/s  69 s as well as the direction of the motion, and, hence,
5 represents velocity.
60 ×
18
Some noteworthy points
1.5 VELOCITY
1. Velocity is a vector while speed is a scalar, both
The speed of an object tells us how fast the object is having same units(m/s).
moving. If the speed is known, one can calculate the 2. If during motion, velocity remains constant
distance traversed by the object in a given time interval. throughout a given interval of time, the motion is said
But to locate the position of the object at the end of the to be uniform.

CORPORATE OFFICE : Parus Lok Complex, Boring Road Crossing, Patna - 1


[8] PHYSICS | STD.IX

3. If velocity is constant, speed will also be constant. Average Velocity


However, the converse may or may not be true, i.e.,
if speed is constant, velocity may or may not be It is the ratio of total displacement to total time taken.
constant as velocity has direction in addition to
magnitude which may or may not change, as in the Total displacement
Average velocity =
case of uniform circular motion. Total time taken
4. Velocity can be positive or negative, as it is a vector; Some noteworthy points :-
but speed can never be negative, as it is magnitude
of velocity. 1. Average speed is a scalar while average velocity is
a vector, both having same units (m/s).
Uniform Velocity
2. Average speed or velocity depends on time interval
When a body has equal displacements in equal intervals over which it is defined.
of time in a specified direction, (howsoever short the time
intervals may be) the body is said to be moving with a 3. For a given time interval, average velocity is single
uniform velocity. valued while average speed can have many values
depending on path followed.
Example: Imagine a car is moving along a straight road
towards east, such that in every one second it has a 4. If after motion, the body comes back to its initial
displacement of 5m. position, then average velocity is zero but average
speed is greater than zero and finite.
East

5. For a moving body average speed can never be


negative or zero (unless t =  ) while average
5m 5m 5m
1s 1s 1s
A B C D velocity can be zero or negative or positive.
In such a case, the uniform velocity of the car is 5 m/s
6. In general, average speed is not equal to magnitude
east as illustrated above.
of average velocity. However, it can be so if the
A body will have a uniform velocity only if : motion is along a straight line without change in
direction.
(i) It has covered equal distances in equal intervals of
time, i.e., the magnitude does not change. Differences between Speed and Velocity
(ii) Its direction remains the same. S. Speed Velo city
No
If any of the two conditions are not fulfilled, then the body .
will not have uniform velocity, but have a variable velocity. 1 D istance travelled by a The distance trav elled by a
body per unit time is body in a particular
Variable Velocity k nown as its speed. direction (i.e.,
displacement) per unit
time is k nown as its
When a body covers unequal distances in equal intervals velocity.
of time in a specified direction or equal distances in equal 2 Average speed of a Av erage velocity of a
intervals of time, but its direction changes, then the body mov ing body cannot be mov ing body can be zero.
z ero.
is said to be moving with a variable velocity.
3 Speed tells how fast a Velocity tells how fast a
body moves. body moves and in which
Example : A car moving along a circular path, such that it
direction it moves.
covers 5 m in every one second, as shown below. 4 Speed is a scalar Velocity is a vector
quantity. quantity.
5 Speed of a body is Velocity of a body c an be
always positive. positive or negativ e.
5m
5m
Illustration 1 : A body travels with velocity v1 for t1 and
1s
1s B with velocity v2 for time t2 in the same direction. Find the
average velocity of the body ?
A C Sol. : Displacement in time (t1 + t2) = v1t1 + v2t2
Certainly, the car has a uniform speed of 5m/s, but its
uniform velocity is not 5 m/s, because the direction of the total displacement  v1t1  v 2 t 2
 Average velocity =
car is changing continuously. total time taken t1  t 2

CORPORATE OFFICE : Parus Lok Complex, Boring Road Crossing, Patna - 1


CBSE | MOTION [9]

Illustration 2 : A body traveling along a straight line


traversed one third of the total distance with a velocity Daily Practice Problem-1
of 4 m/s. The remaining part of the distance was covered
with a velocity of 2 m/s for half the time and with velocity 1. A rabbit moves a distance of 8 cm from P to Q and
of 6 m/s for the other half of time. What is the average then moves a distance of 6 cm at right angles to
velocity over the whole motion ? PQ. What is the magnitude of its displacement ?
Sol. : Let S be the total distance travelled by the body. If
t1 is the time taken to cover the one third distance. 2. A man has to go 50m due north, 40m due east and
20m due south to reach a field from his house.
s/3 s
Then, t1   (a) What distance should he walk to reach the
4 12 field ?
Let t2 be the time for each of the remaining two
journeys. (b) What is his displacement from his house to the
field ?
2s 2s s
Then,  2t 2  6t 2  8 t 2  t 2   3. A car moves 2.00 km towards east, makes a
3 38 12
perpendicular left turn and travels 500 m, makes a
total displacement perpendicular right turn and travels 4.00 km and
 Average velocity  finally stops. Find the displacement of the car.
total time
s s 4. A body covers an arc of a circle of radius ‘r’,
   4 m/s subtending an angle of 120° at the centre of the
t  2t 2  s   s 
1
 12   2  12  circle. What is the magnitude of the displacement of
   
the body ?
Illustration 3 : In one second a particle goes from 5. A car travels the first half of a distance between two
point A to a point B moving in a semicircle (see figure).
places at a speed of 30 km/h and the second half of
What is the magnitude of average velcoity ?
the distance at 50 km/h. Find the average speed of
the car for the whole journey.
1m
A B 6. A car moves for half of its total travel time with a
displacement(AB) speed 80 km/h and for next half of the time at 40
Sol. : Average velocity  km/h. The total distance covered is 60 km. Find the
time
average speed of the car.
2
  2 m/s
1 7. A man travelling in a straight line moves with a
uniform velocity v1 for a certain distance and with a
Illustration 4 : A lecture theatre of a college is 40 ft uniform velocity v2 for the next equal distance. Find
wide and has a door at corner. A teacher enters at 12.00 the average velocity (v).
noon through the door and makes 10 rounds along the
40 ft wall, back and forth during the period and finally 8. There is a square field of side ‘a’ units. A tortoise
leaves the class room at 12.50 pm. through the same starts from one corner and reaches the diagonally
door. Compute his average speed and average velocity. opposite corner of the field in a time interval of t.
Sol. : Total distance travelled in 50 minutes = 800 ft. Find the magnitude of the average velocity of the
tortoise.
800 9. A train travels at a speed of 60 km/h for 0.5 h, at 30
Average speed = ft / min .  16 ft / min .
50 km/h for the next 0.2 h and then at 70 km/h for the
next 0.7 h. What is the average speed of the train ?
At 12.00 noon he is at the door and at 12.50 pm he
is again at the same door. 10. When a person leaves his home for sight-seeing by
his car, the meter reading is 12352 km. When he
Therefore displacement during the 50 min interval
returns home after two hours the reading is 12416
is zero.
km. What is the average speed and average velocity
 Average velocity = zero of the car during this period ?

CORPORATE OFFICE : Parus Lok Complex, Boring Road Crossing, Patna - 1


[ 10 ] PHYSICS | STD.IX

1.6 ACCELERATION Final velocity  Initial velocity


Acceleration 
Time int erval
To develop the idea of acceleration, let us consider a body v u
moving in a straight line with a non-uniform velocity. For  a
t
example, let a train start from rest at station A. When it
Where :
starts moving, its velocity increases and after a certain
time interval it attains a constant velocity. As the next u = Initial velocity of the body
station approaches, its velocity gradually decreases and v = Final velocity of the body
finally becomes zero at the station B. This can be
illustrated as follows. t = Time interval
a = Acceleration of the body
A constant velocity B Units of acceleration
Velocity

g ve
sin loc
rea it yd The C.G..S and S.I units of acceleration are cm/s2 and
inc ec
ity rea
loc sin
m/s2 respectively. At times it is also measured in km/hr2.
ve g
C
Positive Acceleration
O
Station A Station B

These changes in the velocity of a moving body are Acceleration is described by the equation,
d e s c r acceleration. Acceleration is denoted
i b e d i n t e r m s o f

by ‘a’, and is defined as follows: Acceleration,


Final velocity  Initial velocity v  u
The change in the velocity of a body per unit time is a 
called acceleration. Time int erval t

Change in velocity If the final velocity of a moving body is greater than the
Acceleration  initial velocity, i.e., v > u, then,
Time int erval
A positive quantity
If Velocity of a body at t1 = v1 acceleration  = positive quantity
Time
and Velocity of a body at t2 = v2 Thus, the acceleration of a moving body is positive if its
Then, Change in velocity = v2 – v1 final velocity is greater than the initial velocity. In other
words, when the velocity of a body increases with time,
and Time interval = t2 – t1 its acceleration is positive. In common parlance, positive
acceleration is simply called acceleration.
As per definition, the acceleration of the body over the
Example: A body dropped from a certain height gains
 v  v1  velocity as it falls down towards the earth. So, a body
a 2
time interval t1 to t2 is given by
 t 2  t1  falling towards the earth has positive acceleration.
Note : Acceleration is taken to be positive if it acts in the
The acceleration given by the above equation is actually direction of velocity, and negative when it acts in the
the average acceleration over the time interval t1 to t2. direction opposite to the direction of velocity.
However, if the time interval (t2 – t1) is very small, then
acceleration obtained is called acceleration at time t1. Negative Acceleration (or Retardation)
The rate of change of velocity of a body is called its
Final velocity  Initial velocity v  u
acceleration. The change in the velocity may be due to a We know, a  
change in its speed or direction of motion or both. Time int erval t
But when a body moves in a straight line, its direction If the final velocity of a moving body is less than the initial
does not change. So, for bodies travelling along a straight velocity , i.e., v < u, then
line, the acceleration is due to the change in its speed A negative quantity
during its motion. However, when a body moves along a acceleration   negative quantity
Timeinterval
circular path at a constant speed, the acceleration in the
body is due to the change in its direction of motion. Thus, the acceleration of a moving body is negative when
its final velocity is less than the initial velocity. In other
Acceleration of a body in terms of its initial and final words, when the velocity of a body decreases with time,
velocities is given as its acceleration is negative.

CORPORATE OFFICE : Parus Lok Complex, Boring Road Crossing, Patna - 1


CBSE | MOTION [ 11 ]

Example : When a ball (or stone) is thrown vertically it is said to have non-uniform acceleration, or variable
upwards, its velocity decreases with time. So, the acceleration. Acceleration is defined as the rate of change
acceleration of a ball thrown vertically upwards is negative. of velocity. So, the non-uniform acceleration may be
defined as follows:
The velocity of a ball rolling on the floor keeps on
decreasing until it stops. So, the acceleration of a ball When the velocity of a body changes by unequal amounts
rolling on the floor is also negative. in equal intervals of time, it is said to have a non-uniform
acceleration.
Negative acceleration is also called deceleration or
retardation. Example : An auto/car driven on a crowded city-road
with frequent applications of brakes has a non-uniform
So, when the velocity of a body decreases with time, it is acceleration.
said to be under retardation.
1.7 EQUATIONS OF MOTION
For a body undergoing retardation, the final velocity is
less than initial velocity. Thus, retardation is actually There are a variety of quantities associated with the motion
acceleration with a negative sign. For example, if a body of objects, displacement (and distance), velocity (and
has an acceleration of – 5 m/s2, then the retardation of the speed), acceleration, and time. Knowledge of each of
body is +5 m/s2. these quantities provides descriptive information about
an object’s motion. The equations of motion or kinematics
Units of Retardation are a set of equations which can be utilized to determine
unknown information about an object’s motion if other
Retardation (or negative acceleration) has the same units information is known. The equations can be utilized for
as acceleration. Thus, the SI unit of retardation is metre any motion, which can be described as being either a
per second square (m/s2 or ms–2) constant velocity motion (an acceleration of 0 m/s2) or a
constant acceleration motion. They can never be used
What is meant by Uniform Acceleration? over any time period during which the acceleration is
changing. Each of the kinematics equations includes four
W hen a moving body has the same acceleration
variables; if the values of three of the four variables are
throughout its motion, it is said to have uniform
known, then the value of the fourth variable can be
acceleration. The uniform acceleration is also called calculated. In this manner, the kinematics equations
constant acceleration. provide a useful means of predicting information of an
When a body travels in a straight line and its velocity object’s motion if other information is known.
changes by equal amounts in equal intervals of time, it is For example, if the initial and final velocity of a skidding
said to have a uniform acceleration. car is known, (and acceleration) then the displacement
of the car and the time can be predicted using the
The motion of a body with uniform acceleration is called equations of kinematics.
uniformly accelerated motion.
The equations of kinematics which describe an object’s
Examples : motion, are:
(a) A body falling freely under gravity has uniform 1 2
acceleration. 1. v = u + at 2. S = ut + at
2
a
(b) A ball moving down an inclined plane has uniform
acceleration.
3. v2 – u2 = 2as 4. Snth = u+  2n  1
2
A body falling freely Where u = initial velocity
from a certain height A body moving v = final velocity
down an
inclined plane a = acceleration
Inclined plane s = displacement or distance covered
n or t = time in seconds
Ground Note :
Ground
(i) If a body starts from rest, its initial velocity, u = 0
Non-uniform Acceleration (ii) If a body comes to rest (i.e., it stops), its final
velocity, v = 0
Non-uniform acceleration is also called variable
acceleration. W hen a moving body has different (iii) If a body moves with uniform velocity, its
accelerations at different points of time during its motion, acceleration, a = 0

CORPORATE OFFICE : Parus Lok Complex, Boring Road Crossing, Patna - 1


[ 12 ] PHYSICS | STD.IX

Derivations of Equations of Motion 2ut  at2 1 2


or S  or S = ut  at
2 2
v = u + at

v
u
=
2
a
S
1.

2
3.
This equation gives the velocity acquired by a
body in time t. This equation gives the velocity acquired by a
body after it displaces through ‘S’.
Consider a body having initial velocity u. Suppose it
is subjected to a uniform acceleration a so that after Consider a body moving with an initial velocity u
time t its final velocity becomes v. Now, from the having acquired a velocity v after displacing through
definition of acceleration we know that: S. The uniform acceleration of the motion is a.
change in velocity Let the displacement of the body in this time be S.
Acceleration = The displacement of the moving body in time t can
time taken
be found out by considering its average velocity.
(or) Acceleration = Final velocity – Initial velocity
Time taken Since the initial velocity of the body is u and its final
velocity is v, the average velocity is given by:
v–u
So, a =  at = v – u  v = u + at Average velocity
t
By paying due attention to the sign of acceleration, Initial velocity + Final velocity uv
this equation can also be applied to the problems of = 
2 2
uniform retardation. In this case, a will be replaced
by –a. Also,Displacement = Average velocity × Time

u  v   t
1 So, S  ...(i)
2. S = ut + at 2 2
2
And from the first equation of motion, we have:
This equation gives the displacement of the
v u
body in time t. v = u + at  at = v – u or t 
Consider a body moving with an initial velocity u
a
having acquired a velocity v in time interval t, such Substituting this value of t in equation (i), we get:
that a is its uniform acceleration.
Let the displacement of the body in this time be S. u  v v  u v2  u2
The displacement of the moving body in time t can S    v2  u2  2aS
be found out by considering its average velocity.
2 a 2a
a 2

Since the initial velocity of the body is u and its final


n
t h

velocity is v, the average velocity is given by: 4.


Average velocity
This equation gives distance travelled by a body
Initial velocity + Final velocity u  v in nth second.
= 
2 2 Consider a body moving with uniformly accelerated
Also, Displacement = Average velocity × Time motion having acceleration a. The distance of a
particle in time t is given by
u  v   t
So, S  ________________
(1)
2 1 2
S = ut + at , where u = initial velocity, at time t = 0
From the first equation of motion, we have, v = u + 2
at. Substituting this value of v in equation (1), we get

u  u  at   t S
2u  at   t If Sn and Sn – 1 are the distances of the particle in n
and n – 1 seconds, then distance of the particle in
S or
2 2 nth second is, Snth = Sn – Sn–1

CORPORATE OFFICE : Parus Lok Complex, Boring Road Crossing, Patna - 1


CBSE | MOTION [ 13 ]

1 Illustration 3 : A ball is thrown vertically upwards with


Sn = un + a n2 a velocity ‘u’. Calculate the velocity with which it falls to
2 the earth again. (Assume the acceleration of ball is uniform
and it is ‘g’ towards earth)
A C B
Sol. : For a ball thrown vertically upwards,
sn-1 snth
sn Initial velocity = u

Final velocity = v = 0
1
Sn–1 = u(n – 1) + a (n – 1)2
2 For the vertically upward motion, the equation of
motion is
Now,Snth = Sn – Sn–1
v = u – gta
 1   1 2
  un  an2    u n  1  a n  1  So, 0 = u – gta
 2   2 
   
 
1 a u
  un  an2    un  u  n2  1  2n  or ta  ...(i)
 2   2  g

 1   an2 a  For the return journey, when the body falls vertically
  un  an2    un  u    an  downwards, the equation of motion is
 2   2 2 
v = u + gt
1 an2 a a
 un  an2  un  u    an  u  an 
2 2 2 2 Since, u=0
 1
 u  an   Hence v = 0 +gtd
 2
v
td 
 2n  1 a or
g ...(ii)
 u  a   Snth  u  (2n – 1)
 2  2
For same plane of projection, time of ascent = time
Illustration 1 : A car is moving at a speed of 50 km/h. of descent.
After two seconds it is moving at 60 km/h. Calculate the Hence, the velocity of ball with which it falls to earth
acceleration of the car. again is ‘u’.
5 250 Illustration 4 : A particle starts from rest having uniform
Sol. : Here, u = 50 km/h  50  m/s m/s
18 18 acceleration 10 m/s2. Find the displacement covered by
particle in 10 seconds.
5 300
and v  60 km / h  60  m/s m/s Sol. : Given, u = 0
18 18

a = 10 m/s2, t = 10 sec.
1
using S  ut  at2
300 250 50

v u 18  18  50  1.39 m/s2 2
a  18
t 2 2 36
1
S  0(10)   10  102  S = 500 meters
Illustration 2 : A car attains 54 km/h in 20 s after it 2
starts. Find the acceleration of the car.
Illustration 5 : A stone dropped from the top of the
Sol. : u = 0 (as car starts from rest) tower touches the ground in 4 sec. The height of the tower
is about ____________.
5
v  54 km / h  54   15 m/s Sol. : Given, Time, t = 4 s and g = 10 m/s2
18

v u 15  0 1 1
 h  gt   10  (4)  80 m
2 2
As, a   a  0.75 m/s 2
t 20 2 2

CORPORATE OFFICE : Parus Lok Complex, Boring Road Crossing, Patna - 1


[ 14 ] PHYSICS | STD.IX

Illustration 6 : A car starts from rest and accelerates Illustration 7 : A ball is thrown vertically upwards with
uniformly for 10 sec to reach a velocity of 8 m/s. It then a velocity of 20 m/s. How high did the ball go ?
runs at a constant velocity and is finally brought to rest in (Assume the acceleration of ball is uniform and it is 9.8
64 m with a constant retardation. The total distance m
2 towards earth.)
/ s

covered by the car is 584 m. Find the value of


acceleration, retardation and total time taken. Sol. : u = 20 m/s, a = – g = – 9.8 m/s2 (moving against
gravity)
Sol. In the above diagram; car starts from A and
s = ?, v = 0 (at highest point)
achieves velocity 8 m/s at B in 10 sec.
So, for AB, v = 8 m/s, u = 0 m/s, t = 10 sec ; Using, v2 – u2 = 2 as

v = u + at (0)2 – (20)2 = 2 (– g) s
0 m/s 8 m/s 8 m/s 0 m/s – 400 = 2 (– 9.8) s  – 400 = – 19.6 s
A B C D
400
s  s = 20.4 m
 8  0  a  10 19.6
1 Illustration 8 : A particle starts with velocity 10 m/s
 a  0.8 m / s2 and S AB  ut  at 2
2 having acceleration 2 m/s2. Find displacement covered
by body
1
 S AB  0  10   0.8  10 2
2 (a) in 5 second (b) in 5th second.
 S AB  40 m Sol. : u = 10 m/s a = 2 m/s2
After that car runs with constant speed 8 m/s from (a) t = 5 sec.
B to C point. Let that distance be SBC. After that it
comes to rest at point D covering the distance of 1 2
64 m from C to D. using, S  ut  at
2
For C to D : SCD = 64, u = 8 m/s v
= 0 m/s 1
 S  10  5   2  5  75 meters
2

Applying third equation of motion 2


v2 = u2 + 2 as 02 = 82 + 2 × a × 64 a
(b) n = 5 Sn  u  (2n  1)
 a = – 0.5 m/s 2 2
Now, applying first equation of motion 2
 10  (2  5  1) = 10 + 9 = 19 meters.
v = u + at 2
0 = 8 – 0.5 tCD
Daily Practice Problem-2
8
 t CD   16 sec
0.5 1. An object moves with an acceleration of 8ms –2,
starting from rest. Find the distance travelled in one
Now, the total distance, s second.
S = SAB + SBC + SCD
2. A body starts with a velocity of 40 ms –1 and moves
584 = 40 +SBC + 64 with an acceleration of 10 ms –2. Calculate the
distance travelled by it in 15th second.
 SBC = 480 m
3. A train starting from rest and moving with a uniform
 8 × tBC = 480 m acceleration attains a speed of 90 km/h in 5 minutes.
tBC = 60 sec. Find the distance travelled by the train.

So, total time, T = tAB + tBC + tCD 4. A person travelling on a scooter at 43.2 km/h applies
the brakes, giving a deceleration of 6.0m/s2 to his
 T = 10 + 60 + 16 = 86 sec scooter. How far will it travel before stopping?

CORPORATE OFFICE : Parus Lok Complex, Boring Road Crossing, Patna - 1


CBSE | MOTION [ 15 ]

5. A body moving with uniform acceleration covers 100 number of overs is the independent quantity, and the
m in the first 10 seconds and 150 m in the next 10 number of runs scored is the dependent quantity.
seconds. What is the initial velocity of the body?
y – axis
6. A car accelerates from rest at a constant rate  for
some time after which it decelerates at a constant

(No. of runs scored)


Dependent quantity
rate  to come to rest. If the total time taken is t,
find the maximum velocity attained by the car.

7. A car moving on a road with uniform acceleration


covers 20m in the first second and 30m in the next
second. Find the acceleration of the car.
O x – axis
Independent quantity
8. Two cars are travelling towards a junction along two
(No. of overs)
intersecting roads. One of them is 200 m away and
is moving uniformly at 10 m/s. The other, located
100 m away, starts from rest and accelerates
How a Graph is plotted ?
uniformly at 2 m/s2 towards the junction. What is the
Graphs are plotted on specially designed graph papers.
interval between the moments when the cars pass
The whole sheet of the commonly used graph paper is
the intersection?
divided into squares of 1 cm sides. Each side of such
9. A bus accelerates uniformly from rest and acquires squares is further subdivided into millimetres. The lines
a speed of 72 km/hr in 20 seconds. The acceleration showing millimetre markings are generally light in colour.
of the bus is To draw a graph, we generally proceed as follows :-

(A) 10m/s2 (B) 5m/s2 (C) 2m/s2 (D) 1m/s2 (i) Drawing the axes
10. If a body looses half of its velocity on penetrating
3 cm in a wooden block, then how much will it If both the quantities to be plotted are positive, then draw
penetrate more before coming to rest ? two perpendicular lines intersecting each other at one
point. This point of intersection is called the origin. The
(A) 1 cm (B) 2 cm (C) 3 cm (D) 4 cm horizontal axis is called the x-axis, while the vertical axis
is called the y-axis. If one of the two or both the quantities
1.8 GRAPHICAL REPRESENTATION OF are negative, then the axes are drawn so that the origin is
MOTION in the middle of the paper.

y – axis
When you see, you learn faster and understand better.
The data from any measurement can be made more Y
Positive quantity y – axis
informative when presented in the form of a graph. In this
Positive quantity
section, we study the importance of graphs for describing
the motion of a body. Negative quantity Positive quantity
X X
O x – axis
What is a Graph? Positive quantity
x – axis
x – axis (origin)

O Negative quantity
(origin)
A geometrical relationship between two quantities plotted y – axis
The axes are chosen depending upon
on the x-axis and y-axis is called a graphical plot, or simply Y
the values of the quantities to be
a graph. plotted.

One of the two quantities to be plotted can be changed While plotting, it should be remembered that all positive
independently, while the other depends on it. The quantity quantities are plotted on the right and upwards of the
which can be changed independently is called the origin. The negative quantities are plotted on the left and
independent quantity, while the other which depends on downwards of the origin.
it is called the dependent quantity. Generally, the
independent quantity is plotted on the x-axis, and the The independent quantity is plotted on the horizontal
dependent quantity on the y-axis. For example, if we want (OX or OX) axis, and the dependent quantity is plotted
to draw a graph between the number of runs scored and
the number of overs bowled in a cricket match, then the on the vertical (OY or OY ) axis.

CORPORATE OFFICE : Parus Lok Complex, Boring Road Crossing, Patna - 1


[ 16 ] PHYSICS | STD.IX

(ii) Choosing the Scale 1.9 DISPLACEMENT - TIME GRAPHS


The quantities to be plotted on the two axes are generally (i) Displacement-time Graph of a body at Rest
much bigger than the graph paper. So, for each axis, a
convenient scale is chosen so that the whole range of the The position of a body at rest remains unchanged with
quantities can be plotted on a certain length of the axis. time. Let us consider a body at a distance d from a
The choice of scale is made so that the plotted graph reference point in a particular direction. Then from the
satisfies the following conditions :- figure,

(a) The curve or line of the graph should cover the major

Displacement 
portion of the area enclosed by the two axes. A B
s
6 Here, the choice
of scale on x-axis 6
is not proper The choice of both
5 d d
Dependent quantity

5 the scales is proper


Dependent quantity

4 Here, the choice 4


3 of scale on y-axis 3 A B
is not proper
2 2 0 t1 t2
1
Time 
1
0 Displacement – time graph for a body of rest
0 0
0 1 2 3 4 5 6 1 2 3 4 5 6
Independent quantity Independent quantity At time t=0 Displacement = d
(i) Poor choice (ii) Proper choice At time t = t1 Displacement = d
The choice of scales on the two axes: At time t = t2 Displacement = d
(i) a poor choice (ii) a proper choice Thus, there is no change in the displacement of the body
(b) If the plot is a straight line, it should have an with time.
inclination of about 45°, i.e., the graph should lie in For a body at rest, the displacement-time graph is a
the middle of the quadrant enclosed by the two axes straight line parallel to the time-axis. From this graph, one
of the scales. The effect of a good and a bad choice can write
of the scales on the graph is shown in above figure. Velocity of the body at rest
(iii) Plotting of the given Data Displacement during an interval of time
=
The given values of the two quantities are then plotted in Interval of time
the form of points on the graph paper. These points are
dd 0
then joined by a smooth line/curve to get the desired   0
graph.  t 2  t1  t 2  t 1
(iv) Calculations from the Graph Thus the velocity of a body at rest is zero.

The graphs can then be analysed to obtain certain (ii) Displacement-time Graph of a body moving
parameters, such as, the slope and the intercept. with Uniform Velocity
Usefulness of Graphs The displacement-time graph of a body moving with
uniform (constant) velocity is a straight line inclined to the
Graphs are useful in the following ways: time-axis at a certain angle.
(i) A graphical plot can present a huge amount of data
in a compact form. 5

(ii) A graphical plot shows the geometric dependence


Displacement / m 

of one quantity over the other. For example, a graph 3


immediately tells us whether the dependent quantity
2
(on the y-axis) is a linear or non-linear function of
the independent quantity (on the x-axis). 1

(iii) A graphical plot makes the comparison of different 0 10 20 30 40 50

sets of data easier. Displacement–time graph for a body moving with uniform velocity

(iv) A graph can be used to obtain the value of one The slope of the displacement-time graph of a body
quantity for a certain specified value of the other. moving at uniform velocity is equal to the velocity of the
body.

CORPORATE OFFICE : Parus Lok Complex, Boring Road Crossing, Patna - 1


CBSE | MOTION [ 17 ]

Here, the slope of the straight line plot is positive. Let the distance between the house and the post office
Therefore the velocity of the body is positive. be x and time taken to reach post office from his house
be t.
(iii) Displacement-time Graph of a body moving
with an Increasing, Non-Uniform Velocity Note: Remember the distance-time and displacement-
time graphs of a moving body are similar only when the
The displacement-time graph of a body moving with non- body moves along a straight line in its positive direction
uniform velocity curve. without changing its direction.
velocity at B > velocity at A (a) The boy comes back to his house with the same
speed. This means, he takes equal time for the
Displacement 

return journey. Then, the distance-time graph for


the total journey is shown in the figure.
B B

Displacement 
Distance 
A x
A 2x
Time  x
Displacement-time graph for a body moving
0 O
with an increasing non-uniform velocity t 2t Time  t 2t Time 
Here, the slope of the curve increases with time. So, the (b) The boy goes from his house to the post office and
velocity of the body increases with time, then back to his house at the same speed. So, the
i.e., Velocity at B > Velocity at A displacement of the boy at the end of the journey is
zero. The displacement-time graph of the boy’s journey
(iv) Displacement-time Graph of a body moving is shown in the figure.
with Decreasing, Non-Uniform Velocity Note :- The boy is not moving in a straight line (there
is a change of direction). So the displacement-time
The displacement-time graph of a body moving with a
graph is different from the distance-time graph.
decreasing non-uniform velocity is a curve. Here, the slope
of the curve decreases with time. So, the velocity of the
body decreases with time, i.e., Velocity at B < Velocity
1.10 SPEED-TIME GRAPHS OR
at A VELOCITY-TIME GRAPHS
B The geometrical relationship between the velocity of a
Displacement 

body and the time from the start is called a velocity-time


B
graph.

A Velocity at B < velocity at A The shape of the velocity-time graph of a body depends
upon the nature of its motion. For motion along a straight
A
line in a particular direction the slope of a velocity-time
Time  graph is equal to the acceleration of the body.
Displacement-time graph of a body moving
with a decreasing non-uniform velocity
Uses of speed-time or velocity-time graphs of a
(v) Displacement-time Graph of a body straight line motion
moving with a Change of Direction Speed-time or velocity-time graphs can be used for
When a body moves with a change of direction, its
(a) determining the speed or velocity of the body at any
displacement-time graph is not the same as its distance-
particular point of time.
time graph. This is illustrated by taking the following
example. (b) determining the acceleration of the body. The slope
A boy goes from his house to the post office with uniform of a speed-time (or velocity-time) graph equals the
speed. After mailing the letter, he comes back to his house acceleration of the body.
with the same speed. To draw the distance-time and (c) determining the total distance (or displacement)
displacement-time graphs of the boy, let us proceed as travelled by the body in a given time interval.
follows:

CORPORATE OFFICE : Parus Lok Complex, Boring Road Crossing, Patna - 1


[ 18 ] PHYSICS | STD.IX

Calculation of distance travelled from speed- Distance travelled in the time interval t1 to t2
time graphs = AB × AD = Area of the rectangle ABCD
How to calculate the distance travelled in a certain time The area of rectangle ABCD is also called the area under
interval from a speed-time graph ? the graph.
Speed-time graphs can be used for calculating the Thus, the distance travelled by a body in a certain time
distance travelled by the moving body in a certain time. interval is equal to the area under the speed-time graph
The methods of calculating distance from speed-time for the given time interval.
graphs in some typical cases are described below :-
Note: The area of the rectangle under the graph in a
1. When a body moves with a constant speed-time graph is not area in the mathematical sense
of the word. To understand this, let us recall that in
speed in a straight line geometry, the area of a rectangle is given by the product
The speed-time graph for a body moving with a constant of length × breadth and both are measured in units of
speed is a straight line parallel to the time-axis. distance e.g., metre. So, the unit of area is (distance)2
i.e., square metre.
Constant speed, v
In speed-time graphs, the lengths on the two axes do not
B C
Y describe the ‘distance’. In the speed-time graphs, the
length on the x-axis describes the time interval, and the
Speed 

length on the y-axis describes speed. The time interval is


measured in units of time, generally second, and the
speed in units of distance per unit time, generally metre
A D per second. Therefore,
O t1 t2
Area of the rectangle in a speed-time graph is given by,
Time 
Area = Speed × Time
Calculation of the distance travelled by a body
moving at constant speed in the time interval, t2 – t1
 Area  Dis tance  Time = distance
Speed of a body is given by, Time

Distance travelled 2. When a body moves with uniform


Speed  acceleration
Time taken

or, Distance travelled = Speed × Time taken v A

Let us consider a body moving with a constant speed v. If


Speed 

the body travels from time t1 to time t2, then


Area of the
Time interval = (t2 – t1 ) triangle OAB
= Distance
travelled B
Therefore, Distance travelled by the body in the time
O Time  t
interval t1 to t2 = v × (t2 – t1) ...(1) Calculation of the distance travelled by a body
when the speed increases uniformly with time
To get the values of v, t1 and t2 from the speed-time graph
proceed as follows. The above figure shows the speed-time graph of a body
when its speed increases uniformly from zero to v in time t.
Draw perpendiculars from the points A and D on the time-
axis. Let these perpendicular intersect the straight-line Initial speed + Final speed
graph at points B and C respectively. Then, Average Speed 
2
Speed of the body = v = OY = AB = CD
uv 0v v
  
Time, t1 = OA and Time, t2 = OD 2 2 2
or, (t2 – t1) = OD – OA = AD The distance travelled by the body is then given by,

Substituting the values of v and (t2 – t1)in the equation, Distance moved by the body = Average speed × Time
we get v 1
interval   t  vt
2 2

CORPORATE OFFICE : Parus Lok Complex, Boring Road Crossing, Patna - 1


CBSE | MOTION [ 19 ]

From the speed-time graph, v = AB and t = OB 4. When the initial speed is not zero and the
Then, Distance travelled in time, speed increases uniformly
1 1
t   AB  OB  × Perpendicular × Base
2 2
v B
or, Distance travelled in time t = Area of the triangle OAB

Speed 
3. When a body moves with uniform A
retardation u
Area of
trapezium =
Distance travelled C
u A O
0 Time  t

Let the initial speed of the body be u, and let it increase


uniformly to v in time t.
Speed 

Area of the Then, Distance travelled by the body in t


triangle OAB = Average speed × Time interval
= Distance travelled
B
O Time  t uv
But, Average speed =
2
The above figure shows the speed-time graph of a body
when its speed decreases uniformly to zero in time t. Then, uv
the average speed of the body is given by. So, Distance travelled by the body in time t  t
2
Average  Initial speed  Final speed  u  0  u From the speed-time graph,
speed 2 2 2
u = OA ; v = BC ; and t = OC
Then, Substituting these values in above equation, we get
Distance travelled by the body in time t  OA  BC 
t  OC
2
u 1
= Average speed × Time interval  t  ut or, Distance travelled by the body in time t = Area of the
2 2 trapezium OABC
From the speed–time graph, u = OA and t = OB 5. When the speed decreases uniformly and
Substituting the values of v and t in above equation, we the final speed is not zero
get
Let the initial speed of the body be u, and decreases
Distance travelled in time t from the start uniformly to v in time t. Then,
Distance travelled by the body in time t
1 1 = Average speed × Time interval
  OA  OB   Perpendicular × Base
2 2
A (initial speed)
Hence, Distance travelled in time ‘t’ from the start = Area u
of the triangle OAB
Speed 

Note: When the initial or the final speed of a body is zero,


and the speed increases or decreases uniformly, then the v B (final speed)
Area of
area of the triangle formed by the speed-time graph and
trapezium
the time-axis is equal to the distance travelled by the = distance travelled C
moving body in time t. O
0 Time  t

CORPORATE OFFICE : Parus Lok Complex, Boring Road Crossing, Patna - 1


[ 20 ] PHYSICS | STD.IX

u  v  The shaded area gives the distance travelled by the


But, Average speed  body.
2
Distance travelled = Area OAB + Area BCD
t
u  v   t
So, Distance travelled by the body in time
2 A
From the speed-time graph u = OA, v = BC, and t = OC
Substituting these quantities in the above equation, we +
get

Velocity 
Distance travelled by the body in time, t 2t
0
t
 OA  BC  OC O B D
2
or, Distance travelled by the body in time t = Area of the 
trapezium OABC.
C
Note: When the speed of a body increases or decreases
uniformly and its initial or final speed is not zero, then the (b) Velocity – time graph
area of the trapezium formed by the speed time graph for
the given time interval is equal to the distance travelled Displacement = Area OAB + Area BCD
Displacement = Area OAB + Area BCD
by the body in that time interval. = Area
= Area OAB OAB – Area
– Area OABOAB
=0 =0
Speed-time and velocity-time graphs of a body
It should be noted that the distance travelled by the body
thrown vertically upward. is positive while the displacement is zero.
Consider a body thrown vertically upwards. As it goes up,
its speed decreases, becomes zero at the highest point Different types of velocity-time Graphs
and then increases while coming down. The speed-time
graph of such a body is shown in figure. For a straight line motion, there can be six different types
of speed-time (or velocity-time) graphs. These are
described in the figure given in the next page:

Conclusions :

h (i) If velocity–time graph is a straight line but moving


away from velocity time axis, then:

////////////////////////////////// (a) Body is moving with variable velocity.


Upward and downward motion of a body thrown
vertically up from the ground (b) It has uniform acceleration, which can be found
Taking upward direction as the positive direction, the by the slope of graph.
velocity of the body in the upward direction is positive. As
the body moves upwards, its velocity decreases, becomes (c) Displacement can be found by finding area
zero at the highest point, and then becomes negative while under the velocity-time graph.
coming down. The velocity-time graph of such a body is
(d) If slope is positive, then the body has positive
shown in figure.
acceleration and vice–versa.

(ii) If the velocity–time graph is a curve, then:


A C
Speed 

(a) The body has variable velocity and variable


acceleration.

B D (b) Area under the curve represents displacement.


0
O t 2t
Time
(c) Acceleration at any instant can be found by
(a) Speed – time graph finding slope at that point.

The shaded area gives the distance travelled by the body.


Distance travelled = Area OAB + Area BCD
CORPORATE OFFICE : Parus Lok Complex, Boring Road Crossing, Patna - 1
CBSE | MOTION [ 21 ]

Conditions under which Description of the graph Speed –time graph, or


the body m oves Velocity- time graph
1. W hen the speed/velocity Straight line parallel to the tim e -axis Sp eed/velocity of th e
mo ving bo dy rem ains
remains constant (x- axis)

Speed / Velocity
constant

Slope of the speed - time graph gives A B


(slope = 0)
acceleration of th e body. Therefore, the
acceleration of a body m oving with a
constant speed (or velocity) is zero. O Tim e 

2. W hen the initial Straight line sloping upwards and Sp eed/velocity of th e bod y
in creases at a co nstant
speed/velocity is zero, passing through the origin. Slope of the rate (un iformly)
A
and the speed/velocity straight line is positive. Therefore, the slop e
= acceleration
increases uniformly (at a body moving with a uniformly
C B
constant rate) with time. increasing speed/velocity has uniform
acceleration. O Tim e 

3 . W hen the velocity/speed Speed – time or velocity – time graph in


increases non-uniform ly this case is a curve moving upwards.

Speed / Velocity
i.e.,, when the body is Slope of the curve is positive and Speed/velocity
under non -uniform increase s with time. Positive slope of the increases
non-uniformly
acceleration curve at any point is equal to the
acceleration of the body. So, the
slope is + ve
acceleration of the body increases with
time i.e., body is under non - uniform O Tim e 
acceleration

4 . W hen the velocity/ speed Speed -time or velocity - time graph is a Speed/velocity
decreases non -uniform ly curve moving downw ards. Slope of the Decreases
i.e., when the body is curve is negative, and increases with non-uniform ly

Speed / Velocity
under non - uniform time. N egative slope of a speed - time (or Slope is –ve
the body is
retardation velocity- time) curve gives retardation under
(negative acc eleration ) of the body. non-uniform
Here, the retardation increases w ith retardation
time, i.e., the body is under non- uniform O
Tim e 
retardation .
5. W hen the speed/velocity Speed -time or velocity– time graph in a
increases and decreases zig-zag curve. The body is under Speed / velocity changes
Speed / Velocity

irregularly
alternatively, uniform acceleration and retardation
i.e,, the speed/velocity alternatively.
changes alternatively.
O Time 

6. W hen the speed/velocity The speed -time or velocity-time graph


increases in a stepwise looks like a staircase. The body has
manner. alternatively infinitely large acceleration
(vertical motion) and zero acceleration
(horizontal motion). O Tim e 

1.11 ACCELERATION - TIME GRAPHS

Fig(i)
Fig
Fig (i)(i) Fig
FigFig (ii)
(ii) (ii) Fig (iii)
Fig (iii)

CORPORATE OFFICE : Parus Lok Complex, Boring Road Crossing, Patna - 1


[ 22 ] PHYSICS | STD.IX

Figure (i) represents an acceleration–time graph, AB Sol. : (i) Acceleration, a = slope of the
coinciding with time axis. From the figure it is clear
12  8
that acceleration of the body is zero, and hence, it is graph   2 m/s2
moving with a uniform velocity. 42
Figure (ii) represents an acceleration–time graph, B
parallel to time axis. From figure it is clear that as 24
acceleration does not change, therefore body is 20

Velocity
moving with a uniform acceleration and variable 16
velocity. The area of graph, i.e., Acceleration × Time 12
gives change in velocity. 8
4 C
Figure (iii) represents an acceleration–time graph A D
O 2 4 6 8 10 Time(second )
moving away from time as well as acceleration axis.
From the graph it is clear that the body is moving
with variable velocity and variable acceleration. Area (ii) Distance travelled = Area between the line
AB and time axis
of the graph gives change in velocity.
(as direction of velocity does not change) = Area
Illustration 1 : The graph shows the position of a body of triangle ABC + Area of rectangle OACD
at different times. Calculate the speed of the body as it
moves from. 1
  20  10  4  10  Distance = 140 m.
(i) O to P (ii) P to Q and 2
(iii) Q to R
Sol. : Speed of the body = Slope of the curve Illustration 3 : The velocity-time graph of a particle
moving along a straight line is shown in the figure by curve
(i) in region OP OABCD. Calculate the distance covered by the particle
PS 4 between
Speed    1 cm / sec
OS 4 v(m/s)
A
20
15
8 R
B C
10
Distance (cm)

5
P Q D
4 T
O 2 4 6 8 10 12 14 16 18 time(s)

S U V
O
2 4 5 7 (i) t = 0 to t = 18 second
Time(s)

(ii) In region PQ (ii) t = 2 sec. to t = 12 sec. and the maximum


value of acceleration during this interval.
Speed = slope of the PQ = 0 cm/sec.
(iii) In region QR Sol. : (i) Distance covered = Area of OABCDO
1 1
RT 4   8  20  (10  20)  2  4
Speed    2 cm / sec 2 2
QT 2
1
Illustration 2 : A car is moving on a straight road with  10   4  10  170 m
2
uniform acceleration. The velocity of the car varies with
time as follows : (ii) Distance travelled between 2 to 12 sec
1 1
Time (s) 0 2 4 6 8 10   6  (5  20)   2  (20  10)  2  10
2 2
Velocity (m / s) 4 8 12 16 20 24
 125 m
Draw the velocity-time graph. From this graph. Maximum value of acceleration is from 8 sec to
(i) Calculate the acceleration of the car. 10 sec for the part AB of graph.
10  20
(ii) Calculate the distance travelled by the car in 10  Slope of AB   5 m/s2
second. 10  8

CORPORATE OFFICE : Parus Lok Complex, Boring Road Crossing, Patna - 1


CBSE | MOTION [ 23 ]

Displacement 
Daily Practice Problem-3
C
B
1. The graph given below is the distance-time graph of
an object.
Do you think it represents a real situation? Explain. A

0 1 2 3 4 5 6 7
75
B C Time / second
50
5. The velocity time graph of a linear motion is shown
in figure. The displacement from the origin after 8
Distance
in km second is
25

V(m/s)
A 4
0 2 4 6 8 10D 12 14
Time in hours 2
2. In the given figure, velocity of the body at A is 0
1 2 3 4 5 6 7 8 t(sec.)
Displacement

–2
A

(A) 5 m (B) 16 m
(C) 8 m (D) 6 m

O Time B 6. Figure gives the speed time graph of motion of a


car. What is the ratio of the distance travelled by the
(A) zero (B) unity
car during the last two seconds to the total distance
(C) maximum (D) infinite travelled in seven seconds ?
3. Figure shows the displacement (x) - time (t) graph
of a particle moving on the x-axis.
x

t
O
O t0

Choose the correct statement. (A) 1/9 (B) 2/9


(A) The particle is at rest. (C) 1/3 (D) 4/9
(B) The velocity of particle increases up to time t0
7. The velocity-time graph of a body moving in a straight
and then increases.
line is as shown in the figure.
(C) The velocity of particle increases up to time t0
and then becomes constant. Velocity
(m/s) A
(D) The particle moves at a constant velocity up to a 2
time t0, and then stops. E F
1
4. The displacement-time graph of a lift climbing up 0 O B D G
from the ground floor 1 2 3 4 5
-1 Time (s)
to the top of a building is given in the figure alongside.
Using this graph, answer the following: -2
C
(a) What are the states of motion at A, B and C ? W hat is the displacement of the body in five
(b) Plot the velocity-time graph of the lift. seconds?

CORPORATE OFFICE : Parus Lok Complex, Boring Road Crossing, Patna - 1


[ 24 ] PHYSICS | STD.IX

8. Refer figure given below, find the ratio of speed in 10. Acceleration-time graph of a particle moving in a
first two seconds to the speed in the next four straight line is shown in figure. Velocity of particle at
seconds. time t = 0 is 2 m/s. Find the velocity at the end of
fourth second.

2 a (m/s2)
Distance
(m)
1

0 4
1 2 3 4 5 6 Time (s)

9. From the graph find the total distance travelled by a


particle and its average velocity.
v
A t(s)
5m/s O 2 4

B D
0
20 40 t(s)

5m/s
C



CORPORATE OFFICE : Parus Lok Complex, Boring Road Crossing, Patna - 1


CBSE | MOTION [ 25 ]

ADD ON CONCEPT BOOSTER (FOR COMPETITITVE EXAMS.)

1.12 VECTORS
VECTORS: Vectors are those physical quantities, which
have both magnitude and direction and obey the vector law
of addition.
USE OF VECTOR ANALYSIS
Examples : Displacement, velocity, acceleration, force
Suppose a block of mass M is placed on a smooth etc.
horizontal surface. There are two forces F1 and F2 acting
on the block as shown in the figure. Note :- A vector must obey the vector law of addition
otherwise it will not be a vector although having both
magnitude as well as direction.
F2 = 10 N F1 = 5 N
Example: Current has both magnitude and direction, but
it is not a vector. It is a scalar quantity because it does not
obey the vector law of addition, which we will learn in this
Now the question is in which direction will the block move lesson.
and what will be the net force on the block?
REPRESENTATION OF VECTOR
You can answer it easily. The block will move towards left
and net force will be (10 – 5) = 5 N towards left. (i) Geometrical Method : Geometrically a vector is
represented by the directed line segment i.e., by a
Now think of the situation when these forces are neither in line to which a direction has been assigned with an
the same direction nor opposite to each other. arrow-head in the direction of the vector and whose
Suppose F1 and F2 are perpendicular to each other acting length is proportional to the magnitude of the vector.
on the same block as shown in figure.

P
F1 = 5 N


r
F2 = 10 N
We repeat the same question. In which direction will the O
block move and what will be the net force? It will be difficult
to answer. Why? Since you do not know about vectors.
To represent a vector geometrically, a line is drawn
Similar problems will be faced in other physical relations. parallel to the direction of the vector and put an arrow
We will now discuss vectors in detail. on the line along the direction of the vector. Now this
directed line segment, namely, OP as shown in figure
SCALARS AND VECTORS represents the vector in magnitude and direction. It

In the last lesson we have already discussed about
is written as OP . ‘O’ is called the ‘initial point’ of the
physical quantities. All physical quantities have been
categorised in two parts. vector and P, the ‘terminal point’ of it. The vector
   
OP is also written as r i.e., we also write, r = OP .
Physical Quantities
To represent a physical quantity in a vector form, we
put an arrow above the symbol of the physical
quantity. For example, velocity is denoted by v but in
Scalars Vectors

vector form it is represented as v , which is read as
SCALARS : Scalars are those physical quantities, which velocity vector.
have only magnitude but no direction.
Magnitude of vector is called absolute value indicated
Examples : Density, time, temperature, energy, mass, 
distance, speed etc. by | v | (modulus of velocity vector).

CORPORATE OFFICE : Parus Lok Complex, Boring Road Crossing, Patna - 1


[ 26 ] PHYSICS | STD.IX

(ii) Analytical Method : In this method vector is head


represented in terms of unit vector ( iˆ, ˆj and k̂ ), 
b
which we will see in details later on. 
 b
UNIT VECTOR : A unit vector is a vector of unit magnitude 
tail head

and points in a particular direction. It is used to specify the a tail
a
direction only. Unit vector is represented by putting a cap
C
(^) over the quantity.

c
î = Unit vector along positive x-axis 
b

ĵ = Unit vector along positive y-axis A B
a

k̂ = Unit vector along positive z-axis Here we place the vector b in such a way that its tail touch


the head of vector a , which is shown in figure.
The unit vector in the direction of F is denoted by F̂ and
   
 Let AB = a and BC = b
F  
defined by F̂ =   F  | F | Fˆ  
|F| Then the line joining the tail of a and head of vector b ,
  
EQUAL VECTORS : Two vectors are said to be equal if AC gives the sum of vector a and b
 
(i) their magnitudes are equal
Let AC = c
(ii) they are parallel and
     
(iii) they have the same sense of direction. They needn’t Then, ab  c or AB  BC  AC
have the same initial point.
This is known as triangle law of vector addition.
Angle between two vectors means smaller of the two
We can define in this way. “If two vectors are represented
angles between the vectors when they are placed tail to tail
by the two sides of a triangle taken in order, then their
by displacing either of the vectors parallel to itself (i.e.,
resultant or vector sum is represented by the third side of
0     ). the triangle taken in opposite order

 

ANALYTICAL METHOD
B B B
or
  
 
C
A A

A B
(1) (2) (3) 
R
  
Here  represents the angle between A and B . Q
 
1.13 ADDITION OF VECTORS 

O A D
GRAPHICAL METHOD P
 
Triangle Law of Vector Addition : Let us consider two
Let the two vectors P and Q be represented in
 
 
vectors a and b as shown. Now to get the sum of these magnitude and direction by OA and OB respectively..
 
two vectors i.e. ( a + b ), shift any one of the two vectors Proof : considering OA and OB as two adjacent sides,
parallelogram OACB is constructed.
parallel to itself until the tail of one vector is at the head of
another vector (may also use the sliding and free vector We drop a perpendicular CD on OD produced.
properties). In right angled triangle ACD.

CORPORATE OFFICE : Parus Lok Complex, Boring Road Crossing, Patna - 1


CBSE | MOTION [ 27 ]

AD Case IV: When two vectors are of same magnitude


Here, cos  =  AD = AC cos   
AC i.e., (| P |  | Q |)
or, AD = Q cos 
Also, CD = Q sin  R  P 2  P 2  2P 2 cos 
Now in right angle triangle ODC

OC2 = OD2 + DC2  2P 2 (1  cos ) = 2P 2 (2 cos 2 )
2
 OC2  (OA  AD)2  DC2

|R|2 = (P + Q cos)2 + (Q sin)2  R  2P cos
2
= P2 + Q2cos2 + 2PQcos + Q2 sin2
Also,
R  P  Q  2PQ cos 
2 2
 
2 sin cos
 
Let  be the angle, which the resultant R makes with P . P sin  sin  2 2
tan    = 
P  P cos   2 cos 2
DC Q sin  2 cos 2 2
tan  = = 2
OD P  Q cos 

1  Q sin   tan  = tan   = /2
or,   tan   2
 P  Q cos  
Some Special cases SUBTRACTION OF VECTORS
 
Case I : When P and Q are in same direction, Subtraction of vector can be defined in terms of addition
it means  = 0° of two vectors.

R= P 2  Q 2  2PQ cos 0    
P  Q  P  (  Q)
R= (P  Q ) 2 = P + Q
 
In this case resultant is maximum Let P and Q are at an angle  as shown in the figure.
Q sin 0
tan  = =0 
P  Q cos 0 Q

 
Case II : When P and Q are perpendicular to each other,,
Q 
it means  = 90° 
 -Q –

R= P  Q  2PQ cos 90
2 2 P

P
R  P 2  Q2
Q Q 
tan      tan 1     
P P  To get ( P  Q ), first we will draw a vector  Q as shown
  below.
Case III : When P and Q are in opposite direction, it
means  =   
Then angle between P and  Q will be (   )
R= P 2  Q 2  2PQ cos 
 
= P 2  Q 2  2PQ = (P  Q ) 2 = (P – Q) | P  (  Q) |  P2  Q2  2PQcos(   )
In this case resultant will be minimum.
 
Q sin  | P  Q |  P2  Q2  2PQcos 
tan  = =0
P  Q cos 

CORPORATE OFFICE : Parus Lok Complex, Boring Road Crossing, Patna - 1


[ 28 ] PHYSICS | STD.IX

1.14 RESOLUTION OF VECTORS Illustration 1 : There are two vectors having magni-
tude 3 units and 4 units respectively
It is the process of splitting a single vector into two or more
vectors in different directions which together produce the (a) What should be the resultant if angle between them
same effect as is produced by the single vector alone. The is 60° ?
vectors into which the given single vector is splitted are
(b) What should be the angle between them if the
called components of vector. Infact, the resolution of a
magnitude of resultant is
vector is just opposite to composition of vectors.
Let there is force acting on a block, which is on a (i) 1 unit (ii) 5 unit ?
frictionless surface, at an angle  with the horizontal as  
shown in figure. Sol. : (a) | a | = 3 units, | b | = 4 units and  = 60°
  
F R = a + b

 R= a 2  b 2  2ab cos 
M
Now due to this force it will move towards right. = 9  16  2  3  4 cos 60

We want to know the force, which is cause of rightward = 25  12 = 37 units


s
motion.
It can be known by the components of the force F. The (b) (i) R  a2  b2  2  3  4 cos 
cause of horizontal motion is horizontal component which
will be F cos  .  1  9  16  2  3  4 cos 
We can understand these things in this way.
 1 = 25 + 24 cos 
Let force F is acting from P to Q
We want to know the force in the direction PR and PS. 24
 cos  = = –1
24
Q
S
   = 
 F
F sin (ii) (5)2 =  25  24 cos  

P
 R  25 = 25 + 24 cos 
F cos
0 = 24cos   cos  = 0
In right angle  PQR
or,  =  /2
PR FPR
cos  = PQ  F  PR = PQ cos  Illustration 2 : What is the angle between two
PQ
     
FPR  Fcos  vectors A and B , if | A + B | = | A – B | ?

So, horizontal component  F cos θ Sol. : We know that


 
In right angle  PQR | A  B|  [A 2  B2  2AB cos ] ,
RQ
sin    
PQ | A  B |  [A 2  B 2  2AB cos ]

 RQ  PQ sin  and according to question


  
 FPS  FRQ  FPQ sin  [A 2  B 2  2AB cos ]  [A 2  B 2  2AB cos ]

So, vertical component = F sin  which on solving gives, cos  = 0   =  /2


 
In this way we resolve the vector in two perpendicular i.e., the vectors A and B are perpendicular to
directions. each other.

CORPORATE OFFICE : Parus Lok Complex, Boring Road Crossing, Patna - 1


CBSE | MOTION [ 29 ]

Illustration 3 : A mass of 2 kg lies on a plane making


an angle 30° to the horizontal. Resolve its weight along and Daily Practice Problem-4
perpendicular to the plane. Assume g = 10 m/s2.
1. Which one of the following is not a vector ?
(A) displacement (B) work
(C) force (D) gravitational field.
2. Which one of the following is not a scalar ?
(A) time (B) length
mg cos  (C) mass (D) weight.
mg sin  
3. Let A and B be the two vectors of magnitude 10
unit each. If they are inclined to the x-axis at angles
mg  30° and 60° respectively, the magnitude of the
resultant will be
Sol. : As shown in the figure, the component of weight (A) 20 cos 15° (B) 20 cos 60°
along the plane = mg sin  = 2 × 10 × sin 30° = 10 N. (C) 20 cos 30° (D) 30 cos 60°
 
The component of weight perpendicular to plane = 4. A vector A makes an angle of 20° and B makes an
angle of 110° with the x-axis. The magnitudes of
mg cos 30° = 2 × 10 × 3 / 2 = 17.3 N. these vectors are 3 m and 4 m respectively. The
magnitude of the resultant will be
 (A) 25 m (B) 15 m (C) 10 m (D) 5 m
Illustration 4 : If P = 2 iˆ  3jˆ – 2 k̂ and 5. What is the angle between two vector forces of equal
 magnitude such that the resultant is one-third as
Q = 4iˆ  2jˆ  kˆ , then calculate the magnitude of much as either of the original forces ?

     17  1

(a) P  Q

(b) P  Q (c) 2 P  Q (A) cos–1   18  (B) cos–1  3 
   
(C) 45º (D) 120º
  
     

(d) P P
–QQ
2
6. If A  B  C and the magnitude of A, B, C are 5,
   
Sol. : (a) P  Q  (2iˆ  3 ˆj  2kˆ )  ( 4iˆ  2 ˆj  kˆ ) 4, 3 units then angle between A and C is-
(A) sin–1 (3/4) (B) cos–1(3/5)
= 6iˆ  ˆj  kˆ , 3
  (C) cos–1(4/5) (D) sin–1  
5
PQ = 36  1  1 = 38
7. Find the magnitude of resultant of following three
 
forces acting on a particle.
(b) P Q  (2iˆ  3 ˆj  2kˆ )  ( 4iˆ  2 ˆj  kˆ ) 
F1  20N in eastward direction,

 2iˆ  3 ˆj  2kˆ  4iˆ  2 ˆj  kˆ   2iˆ  5 ˆj  3kˆ F2  20N due north east and
  
PQ = 4  25  9 = 38 F3  20N in southward direction

  8. If the angle between the unit vectors â and b̂ is


(c) 2 P  Q
60°, then | â  b̂ | is
= ( 4iˆ  6 ˆj  4kˆ )  ( 4iˆ  2 ˆj  kˆ )  8iˆ  4 ˆj  3kˆ (A) 0 (B) 1 (C) 2 (D) 4
  9. An aeroplane takes off at an angle of 30º to the
 2P  Q = 64  16  9 = 89 horizontal runway. The component of its velocity
along the runway is 200 km h–1. What is the actual
 
(d) P  2Q  (2 ˆi  3 ˆj  2k)
ˆ  (8 ˆi  4 ˆj  2k)
ˆ velocity of the plane ? What is the vertical component
of its velocity?
  6 ˆi  7 ˆj  4 kˆ 10. Two forces of 12N and 9N are acting at a point. The
  ratio of the magnitude of maximum and minimum
 P  2Q  36  49  16  101 resultant is :
(A) 9 : 12 (B) 21 : 11 (C) 7 : 1 (D) 1 : 7

CORPORATE OFFICE : Parus Lok Complex, Boring Road Crossing, Patna - 1


[ 30 ] PHYSICS | STD.IX

PROJECTILE MOTION
1.15 PROJECTILE Different case of projection are shown in the figure
below.
If an object is given an initial velocity in any direction
and then allowed to travel freely under gravity, then the
object is called projectile.
Examples of projectiles are :
(i) A body dropped from the window of a moving train.
(ii) A bullet fired from a rifle.
(iii) A piece of stone thrown in any direction.
(iv) A Javelin or hammer thrown by an athlete. GROUND TO GROUND PROJECTION
(v) A bomb released from an aeroplane in flight. Let us consider a particle which is projected with initial
MOTION UNDER GRAVITY velocity u at an angle  with horizontal (called angle of
projection). The velocity u has two rectangular
The acceleration with which a body travels under gravity components:
is called acceleration due to gravity ‘g’. Its value is 9.8
m/s 2 (or  10 m/s 2). If you have to take g = 10 m/s 2 (i) The horizontal com ponent, u x =u cos  which
then it must be mentioned in the question otherwise remains constant throughout the motion of particle.
take g = 9.8 m/s 2.
(ii) The vertical component, u y= u sin  , which change
(i) If the body moves upwards (or thrown up) g is taken
negative (i.e. motion is against gravitational pull of with time due to the effect of gravity.
earth). So equation of motion becomes.

1 2 2
v  u  gt, s  ut  gt , v  u2  2gh
2
(ii) If a body travels downward (towards earth) then g
is taken +ve. So equations of motion becomes

1 2 2
v  u  gt, s  ut  gt , v  u2  2gh
2
(iii) If a body is projected vertically upwards with certain
velocity then it returns to the same point of
projection with the same velocity in the opposite
direction.

PROJECTILE MOTION
The motion of projectile is called projectile motion. It is
a two dimensional motion. It is considered as the
combination of two simultaneous motion in mutually
perpendicular direction, which are independent from
each other.
Assumption used in projectile motion :
(i) There is no air resistance on the projectile.
(ii) The effect due to curvature of the earth is negligible.
(iii) The effect of the earth rotation is negligible.
(iv) For all points of the trajectory, the acceleration due
to gravity is constant both in magnitude and
direction.

CORPORATE OFFICE : Parus Lok Complex, Boring Road Crossing, Patna - 1


CBSE | MOTION [ 31 ]

(a) Velocity at any time t : (e) Horizontal Range (R) :


Using first equation of motion in vertical direction, Maximum horizontal distance travelled by the
we have pr oj ec tile is k nown as r ange. Range c an be
vy = uy – gt = u sin  – gt calculated by multiplying horizontal component of
 velocity by time of flight.
 Velocity at any time t, v  v x ˆi  v y ˆj
2uy
 R  ux  T  ux 
v  u cos  ˆi  (u sin   gt)jˆ g
[ since vx  ucos  ] 2uxuy 2u cos  · u sin 
 
(b) Velocity at any height : g g
At any height h,
u2 sin 2
or R
v x  u x  u cos  ...(i) g
and v y  uy  2gh  (u sin )2  2gh
2 2
...(ii) For a given value of u, range will be maximum when
Squaring (i) and adding with equation (ii), we get sin 2 is maximum.

v 2x  v 2y  u2 cos2   u2 sin2   2gh So for maximum range, sin 2  = 1 or 2  = 900


or  = 45°.
 u  2gh
2
[Since sin   cos   1]
2 2
u2
 Thus, Rmax  and Corresponding
 | v | v 2x  v 2y  u2  2 gh g

 v  u2  2gh u2 sin2 45 u2


maximum height, H  
2g 4g
(c) Time of Flight (T) :
The displacement in vertical direction (y-axis) Illustration 1 : A body is projected with a speed of
becomes zero in whole time of motion (T). So we 30 ms –1 at an angle of 30º with the vertical. Find the
have maximum height, time of flight and the horizontal range
of the motion. [ Take g = 10m/s 2 ]
1 1 2
y  uy t  ay t2 or 0  uy T  gT Sol.: Here u = 30 ms –1,
2 2
2u y 2u sin  Angle of projection,  = 90 – 30 = 60º
which gives T  
g g
u2 sin2  30 2 sin2 60º
(d) Maximum Height Attained (H) : Maximum height, H = =
2g 10
At the highest point vy = 0, so with the help of third
equation of motion we have :
900 3 135
=  = m
20 4 4

2u sin  2  30  sin60
Time of flight,T = =
g 10

= 3 3 sec.

Horizontal range,
v 2y  u2y  2gh or 0  u 2y  2gH
u2 sin  302 sin120
R   77.9 m
u2y u2 sin2  g 10
which gives H  
2g 2g

CORPORATE OFFICE : Parus Lok Complex, Boring Road Crossing, Patna - 1


[ 32 ] PHYSICS | STD.IX

Illustration 2 : The maximum range of a gun on 1.16 CIRCULAR MOTION


horizontal terrain is 16 km. If g  10 m / s . What must
2
When a particle moves in a plane such that its distance
be the muzzle velocity of the shell ? from a fixed (or moving) point remains constant, then
(A) 200 m/s (B) 400 m/s its motion is known as circular motion with respect to
that fixed (or moving) point. The fixed point is called
(C) 100 m/s (D) 50 m/s centre, and the distance of particle from it is called
u2 radius.
Sol. : (B) R
max   16  103  u  400m / s
g KINEMATICS OF CIRCULAR MOTION
Illustration 3 : A body is thrown horizontally from
the top of a tower of height 5 m. It touches the ground Variables of Motion :
at a distance of 10 m from the foot of the tower. The (a) Angular Position :
initial velocity of the body is (g = 10 ms –2)
(A) 2.5 ms –1 (B) 5 ms –1 To decide the angular position of a point in space we
need to specify (i) origin and (ii) reference line. The
(C) 10 ms –1 (D) 20 ms –1 angle made by the position vector w.r.t. origin, with the
2h reference line is called angular position. Clearly angular
Sol. : (C) R  u   10  u 2  5  u  10m / s position depends on the choice of the origin as well as
g 10 the reference line. Circular motion is a two dimensional
Illustration 4 : An aeroplane moving horizontally motion or motion in a plane.
with a speed of 720 km/h drops a food packet, while
flying at a height of 396.9 m. The time taken by a food
packet to reach the ground and its horizontal range is
(Take g = 9.8 m/sec 2)
(A) 3 sec and 2000 m (B) 5 sec and 500 m
(C) 8 sec and 1500 m (D) 9 sec and 1800 m
Sol.: (D) Given u 720km/hr 200m/s
Suppose a particle P is moving in a circle of radius r
2h 2  396.9 and centre O.
t   9 sec
g 9.8
The angular position of the particle P at a given instant
 R  u  t  200  9  1800m may be described by the angle  between OP and OX.
Illustration 5 : A particle is projected up from the This angle  is called the angular position of the
ground with a velocity of 20 m/s at an angle of projec- particle.
tion 30° with the horizontal. Find [Take g = 10 m/s 2]
(b) Angular Displacement :
(a) the time of flight,
(b) the greatest height reached and Definition : Angle through which the position vector of
the moving particle rotates in a given time interval is
(c) the horizontal range called its angular displacement. Angular displacement
Sol. : (a) The time of flight, depends on origin, but it does not depends on the
reference line. As the particle moves in a circular path
2u sin  2  20  sin 30 its angular position  changes. Suppose the point
T   2s
g 10
rotates through an angle  in time t , then  is
(b) The greatest height reached, angular displacement.
u2 sin2  (20 / 2)2 • Important points :
H  5 m
2g 2  10
(i) Angular displacement is a dimensionless quantity.
(c) The horizontal range, Its SI unit is radian, some other units are degree
and revolution
u2 sin 2 20 2  sin 60
R   34 m 2  rad = 360° = 1 rev
g 10

CORPORATE OFFICE : Parus Lok Complex, Boring Road Crossing, Patna - 1


CBSE | MOTION [ 33 ]

(ii) Direction of small angular displacement is decided Motion with constant angular acceleration
by right hand thumb rule. When the fingers are
directed along the motion of the point then thumb Circular motion with constant angular acceleration is
will r epr es ent the dir ec tion of angular analogous to one dimensional translational motion with
displacement. constant acceleration. Hence even here equation of
motion have same form.
(c) Angular Velocity  :
 0  Initial angular velocity
Angular displacement
av 
Total time taken   Final angular velocity
2  1    Constant angular acceleration
 av  t  t  t
2 1
  Angular displacement
where 1 and  2 are angular position of the particle at
(i)   0   t
time t1 and t2. Since angular displacement is a scalar,
average angular velocity is also a scalar. 1
(ii)   0 t  t 2
• Important points : 2
(i) Angular velocity has dimension of [T –1] and SI unit
   0 
rad/s. (iii) 2  02  2 ,    t
 2 
(ii) For a rigid body, as all points will rotate through
same angle in same time, angular velocity is a 
characteristic of the body as a whole, e.g., angular (iv) nth  0 
2
 n   n  1 
velocity of all points of earth about earth’s axis is
(2 / 24) rad/h. Directions of angular displacement, angular velocity and
angular acceleration:
(iii) If a body makes ‘N’ rotations in ‘t’ seconds then
average angular velocity in radian per second will
be

2N
av 
t
(iv) If T is the time period and ‘f’ is the frequency of
2
uniform circular motion, then av   2f RADIAL AND TANGENTIAL ACCELERATION :
T
(v) Direction of angular velocity is along the axis of There are two types of acceleration in circular motion;
rotation and decided by right hand thumb rule. Tangential acceleration and centripetal acceleration.

(d) Angular Acceleration : (a) Tangential Acceleration :

Let 1 and 2 be the instantaneous angular speeds Component of acceleration directed along tangent of
circle is called tangential acceleration. It is responsible
at times t1 and t2 respectively, then the average angular
for changing the speed of the particle. It is defined as,
acceleration  av is defined as
   v
   1  at   Rate of change of speed  at = r
av  2  t
t 2  t1 t
• Important Points :
(i) Direction of angular acceleration is along the axis
of rotation. (i) If tangential acceleration is directed in direction
of velocity then the speed of the particle increases.
(ii) If   0, circular motion is said to be uniform. For
motion with constant angular velocity (ii) If tangential acceleration is directed opposite to the
direction of velocity then the speed of the particle
  t,   0 decreases.

CORPORATE OFFICE : Parus Lok Complex, Boring Road Crossing, Patna - 1


[ 34 ] PHYSICS | STD.IX

(b) Centripetal Acceleration : Illustration 3 : A fan is rotating with angular velocity


It is responsible for change in direction of velocity. In 100 rev/sec. Then it is switched off. It takes 5 minutes
c irc ular m otion, ther e is always a c entr ipetal to stop. (a) find the total number of revolution made
acceleration. before it stops. (Assume uniform angular retardation)
Centripetal acceleration is always variable because it (b) Find the value of angular retardation (c) Find the
changes its direction. average angular velocity during this interval.

Centr ipetal ac celer ation is also c alled r adial    0   100  0 


acceleration or normal acceleration. Sol.: (a)     t   2   5  60
 2   
v2
ac  ar   15000 revolution.
r
(c) Total Acceleration :
Total acc eleration is vector sum of centripetal (b)   0  t
acceleration and tangential acceleration.
   1
a  ar  a t  0  100  (5  60)    rev / sec
or a  a2t  ar2 3
Total Number of Revolution
at (c) ωav =
 Total time taken
O
15000
  50 rev/sec
5  60
at
Also, tan  
ar Illustration 4 : Find the time period of meeting of
minute hand and second hand of a clock.
Illustration 1 : :Is the angular velocity of rolation of
hour hand of a watch greater or smaller than the angular 2 2
velocity of Earth’s rotation about its own axis? Sol. : min  rad / min, sec  rad/min
60 1
Sol. : Hour hand completes one rotation in 12 hours
 sec  min  2  (for second and minute hand
while Earth completes one rotation in 24 hours.
to meet again)
So, angular velocity of hour hand is double the
angular velocity of earth.

 2 
  
 T 
Illustration 2 : A particle is moving with constant
speed in a circular path. Find the ratio of average
velocity to its instantaneous velocity when the particle

describes an angle   .
2
Sol. : Time taken to describe angle , ( sec  min ) t  2   2(1  1/ 60)t  2
 R R
t  60
 v 2v  t min.
59
Average  Total displacement  2R

2 2
v
velocity Total time R / 2v  Illustration 5 : A particle travels in a circle of radius
20 cm at a speed that uniformly increases. If the speed
changes from 5.0 m/s to 6.0 m/s in 2.0 s, find the
Instantaneous velocity = v angular acceleration.
The ratio of average velocity to its instantaneous
Sol.: Since speed increases uniformly, average
2 2
velocity  tangential acceleration is equal to instantaneous
 tangential acceleration.

CORPORATE OFFICE : Parus Lok Complex, Boring Road Crossing, Patna - 1


CBSE | MOTION [ 35 ]

 The instantaneous tangential acceleration is 5. A projectile fired with initial velocity u at some angle
given by  has a range R. If the initial velocity be doubled
at the same angle of projection then, the range
v 2  v1 6.0  5.0
at   m/s2  0.5 m/s2 will be :
t 2  t1 2.0
(A) 2R (B) R/2 (C) R (D) 4R
The angular acceleration is
6. If a body A of mass M is thrown with velocity v at
0.5m/s2 an angle 30° to the horizontal and another body B
  at / r   2.5 rad/s2
20cm of same mass is thrown at an angle of 60° to the
horizontal, the ratio of range of A and B will be :
Illustration 6 : The moon orbits the earth with a
period of 27.3 days at a distance of 3.84  108 m from (A) 1 : 3 (B) 3 : 1 (C) 1 : 3 (D) 1 : 1
the centre of earth. Find its linear speed and centripetal
acceleration. 7. At what angle to the horizontal should an object
Sol.: The period of revolution of moon , be projected so that the maximum height reached
is equal to the horizontal range ?
T = 27.3 days = 2.36 × 10 6 s
2 (A) tan  = 2 (B) tan  = 4
Linear speed , v = R = R
T (C) tan  = 2/3 (D) tan  = 3
2  3.14  3.84  108
= 1021.83 ms –1 8. A motor car is travelling at 20 m/s on a circular
2.36  10 6
road of radius 100 m. It is increasing its speed at
v2 (1021.83)2 the rate of 3 m/s 2. Acceleration of motor car is :
Centripetal acceleration = =
R 3.84  108 ( A ) 3 m / s e c
2 (B) 5 m/sec 2
–3 –2
= 2.72 × 10 ms
(C) 8 m/sec 2 (D) None of these

Daily Practice Problem-5 9. The rear wheels of a car are turning at an angular
speed of 60 rad/s. The brakes are applied for 5s,
1. A body is projected with an angle  . The maximum causing a uniform angular retardation of 8 rad/s 2.
height reached is h. If the time of flight is 4s and The number of revolutions turned by the rear
g = 10 m/s 2, then value of h is wheels during the braking period is about
(A) 40 m (B) 20 m (C) 5 m (D) 10 m (A) 48 (B) 96 (C) 32 (D) 12
2. An arrow is shot into air. Its range is 200 m and its
time of flight is 5s. If g = 10 m/s 2 , then horizontal 10. If the length of the second’s hand in a stop clock
component of velocity of the arrow is is 3 cm, the angular velocity and linear velocity of
the tip is
(A) 12.5 m/s (B) 25 m/s
(C) 31.25 m/s (D) 40 m/s (A) 0.2047 rad/sec., 0.0314 m/sec
3. A missile is fired for maximum range with a initial (B) 0.2047 rad/sec., 0.0314 m/sec
velocity of 20 ms –1. If g = 10 ms –2, the range of
missile is (C) 0.1472 rad/sec., 0.06314 m/sec
(A) 50 m (B) 60 m (C) 20 m (D) 40 m (D) 0.1047 rad/sec., 0.00314 m/sec
4. At the top of the trajectory of a projectile, the
11. What is the angular velocity of earth ?
directions of its velocity and acceleration are :
(A) Perpendicular to each other
2 2
(B) Parallel to each other (A) rad / sec (B) rad / sec
86400 3600
(C) Inclined to each other at an angle of 45°
2 2
(D) Antiparallel to each other (C) rad / sec (D) rad / sec
24 6400

CORPORATE OFFICE : Parus Lok Complex, Boring Road Crossing, Patna - 1


[ 36 ] PHYSICS | STD.IX

12. When a body moves with a constant speed along Relative velocity in one dimension :
a circle,
If x A is the position of A w.r.t. ground, x B is position of B
(A) no work is done on it.
w.r.t. .ground and x AB is position of A w.r.t. B then we
(B) no acceleration is produced in the body can say vA = velocity of A w.r.t. ground
(C) no force acts on the body vB= velocity of B w.r.t. ground
(D) its velocity remains constant
and vAB = velocity of A w.r.t. B
13. In case of uniform circular motion which of the
f ollowing phys ic al quantity do not r em ain Thus vAB = vA – vB
constant ?
Note :- vBA = – vAB
(A) Speed (B) Momentum
(C) Kinetic energy (D) Mass Illustration 1 : An object A is moving with 5 m/s and
B is moving with 20 m/s in the same direction. (Positive
14. An electric fan has blades of length 30 cm as x-axis)
measured from the axis of rotation. If the fan is
rotating at 1200 r.p.m. The acceleration of a point (i) Find velocity of B with respect to A.
on the tip of the blade is about
(ii) Find velocity of A with respect to B
(A) 1600 m/sec 2 (B) 4740 m/sec 2
Sol.: (i) vB= +20 m/s ; vA = + 5 m/s
(C) 2370 m/sec 2 (D) 5055 m/sec 2
15. The acceleration of a train travelling with speed vBA = vB – vA = + 15 m/s
of 400 m/s as it goes round a curve of radius 160 (ii) vB = + 20 m/s, v A = + 5 m/s
m, is
vAB = vA – vB = –15 m/s
(A) 1 km/s 2 (B) 100 km/s 2
(C) 10 km/s 2 (D) 0.1 km/s 2 Illustration 2 : A particle A is moving with a speed
of 10m/s towards right and another particle B is moving
1.17 RELATIVE MOTION : at speed of 12 m/s towards left. Find their velocity of
approach.
Motion is a combined property of the object under study
as well as the observer. It is always relative, there is no
such thing like absolute motion or absolute rest. Motion
is always defined with respect to an observer or
reference frame. Sol.: vA = +10 m/s, vB = – 12 m/s,
Relative Motion in one Dimension : vAB = vA – vB = 10 – (–12) = 22 m/s
Relative Position : It is the position of a particle w.r.t. since separation is decreasing hence
observer.
vapp = |vAB| = 22m/s.
In general if position of A w.r.t. origin is x A and that of B
w.r.t. origin is x B then “Position of A w.r.t B” ( x AB) is Illustration 3 : A particle A is moving with a speed
X AB =X A – X B. of 20 m/s towards right and another particle B is moving
at a speed of 5m/s towards right. Find their velocity of
approach.

Relative Velocity :
Sol.: vA =+20, vB = +5
Definition : Relative velocity of a particle A with respect
to B is defined as the velocity with which A appears to  vAB = vA – vB = 20 – (+5) = 15m/s
move if B is considered to be at rest. In other words, it
is the velocity with which A appears to move as seen since separation is decreasing hence
by B considering itself to be at rest. vapp = |vAB| = 15m/s

CORPORATE OFFICE : Parus Lok Complex, Boring Road Crossing, Patna - 1


CBSE | MOTION [ 37 ]

2dv dv
Daily Practice Problem-6 (A)
v  u2
2 (B)
v  u2
2

1. An observer in a car P moving towards north with 3dv


speed v observes that another car Q is moving (C) (D) None of these
v  u2
2
towards east with the same speed v. The true
velocity of Q is : 6. A bus is beginning to move with an acceleration
of 1 m/s 2. A boy who is 48 m behind the bus starts
(A) v towards east running with constant speed of 10 m/s. The earliest
time when the boy can catch the bus is
(B) v 2 towards north-east
(A) 8 sec (B) 10 sec (C) 12 sec (D) 14 sec
(C) v towards south-east
7. Two cars C1 and C2 are approaching each other
(D) v 2 towards south-west on a straight track, C 1 has velocity 5 m/s and C 2
has 3 m/s. Their relative velocity will be
2. A particle P is moving towards North direction with
velocity 5 m/s. Another particle moves along the (A) 2 m/s (B) 8 m/s
same direction with velocity 10 m/s. Find the (C) 4 m/s (D) 16 m/s
velocity of P with respect to Q
 
(A) 15 m/s (B) 5 m/s (C) 3 m/s (D) 4 m/s 8. v A  (xiˆ  6j)
ˆ m/s and vB  (3iˆ  2j)
ˆ m/s, find x
3. In the previous question if Q is moving towards such that the relative speed of A with respect to B
south what will be the velocity of Q with respect becomes 5 m/s
to P ? (A) 3 (B) 4 (C) 5 (D) 6
(A) 5 m/s (B) 3 m/s (C) 14 m/s (D) 15 m/s 9. The velocity of particle P due east is 4 m/s and
4. Two balls A and B are falling freely from a tower. that of Q is 3 m/s due north. What is the velocity
What will be the relative acceleration of A with of P w.r.t. to Q ?
respect to B ? 10. An object A is moving with 10 m/s and B is moving
(A) 0 m/s 2 (B) 10 m/s 2 with 5 m/s in the same direction of positive x-axis.
A is 100 m behind B as shown. Find time taken by
(C) 5 m/s 2 (D) 15 m/s 2 A to meet B.
5. A swimmer capable of swimming with velocity ‘v’ 10m/s 5m/s
relative to water jumps in a flowing river having
velocity ‘u’. The man swims a distance d down A B
stream and returns back to the original position.
Find out the time taken in complete motion. 100 m distance



CORPORATE OFFICE : Parus Lok Complex, Boring Road Crossing, Patna - 1


[ 38 ] PHYSICS | STD.IX

NCERT SECTION
PRACTICE CORNER
NCERT INTEXT EXERCISE 15. What is the quantity which is measured by the area
occupied below the velocity–time graph?
1. An object has moved through a distance. Can it have
zero displacement? If yes, support your answer with 16. A bus starting from rest moves with a uniform
an example. acceleration of 0.1 ms –2  for 2 minutes. Find
(a) the speed acquired, (b) the distance travelled.
2. A farmer moves along the boundary of a square field
of side 10 m in 40 s. What will be the magnitude of 17. A train is travelling at a speed of 90 kmh–1. Brakes are
displacement of the farmer at the end of 2 minutes applied so as to produce a uniform acceleration of
20 seconds? –0.5 ms–2. Find how far the train will go before it is
brought to rest.
3. Which of the following is true for displacement?
18. A trolley, while going down an inclined plane, has an
(a) It cannot be zero. acceleration of 2 cms–2. What will be its velocity 3 s
(b) Its magnitude is greater than the distance after the start ?
travelled by the object. 19. A racing car has a uniform acceleration of 4 ms–2. What
4. Distinguish between speed and velocity. distance will it cover in 10 s after start ?

5. Under what condition(s) is the magnitude of average 20. A stone is thrown in a vertically upward direction with a
velocity of an object equal to its average speed? velocity of 5 ms–1. If the acceleration of the stone during
its motion is 10 ms–2 in the downward direction, what
6. What does the odometer of an automobile measure? will be the height attained by the stone and how much
7. What does the path of an object look like when it is in time will it take to reach there ?
uniform motion?
NCERT TEXT BOOK EXERCISE
8. During an experiment, a signal from a spaceship
reached the ground station in five minutes. What was 1. An athlete completes one round of a circular track
the distance of the spaceship from the ground station? of diameter 200 m in 40 s. What will be the distance
The signal travels at the speed of light, that is, covered and the displacement at the end of 2
3 × 108 m s–1. minutes 20 s ?

9. When will you say a body is in (i) uniform acceleration? 2. Joseph jogs from one end A to the other end B of a
(ii) non-uniform acceleration? straight 300 m road in 2 minutes 30 seconds and
then turns around and jogs 100 m back to point C
10. A bus decreases its speed from 80 kmph to in another 1 minute. What are Joseph’s average
60 km h–1 in 5 s. Find the acceleration of the bus. speeds and velocities in jogging (a) from A to B
11. A train starting from a railway station and moving with and (b) from A to C ?
uniform acceleration attains a speed of 40 km h–1 in 3. Abdul, while driving to school, computes the
10 minutes. Find its acceleration. average speed for his trip to be 20 kmh –1. On his
12. What is the nature of the distance–time graphs for return trip along the same route, there is less traffic
uniform and non-uniform motion of an object? and the average speed is 40 kmh –1. What is the
average speed for Abdul’s trip ?
13. What can you say about the motion of an object whose
distance-time graph is a straight line parallel to the 4. A motorboat starting from rest on a lake accelerates
time axis? in a straight line at a constant rate of 3.0 ms –2 for
8.0 s. How far does the boat travel during this time ?
14. What can you say about the motion of an object if its
speed-time graph is a straight line parallel to the time 5. A driver of a car travelling at 52 kmh –1 applies the
axis? brakes and accelerates uniformly in the opposite

CORPORATE OFFICE : Parus Lok Complex, Boring Road Crossing, Patna - 1


CBSE | MOTION [ 39 ]

direction. The car stops in 5 s. Another driver going 7. A ball is gently dropped from a height of 20 m. If
at 3 kmh–1 in another car applies his brakes slowly its velocity increases uniformly at the rate of
and stops in 10 s. On the same graph paper, plot 10 ms–2, with what velocity will it strike the ground?
the speed versus time graphs for the two cars. After what time will it strike the ground?
Which of the two cars travelled farther after the
brakes were applied ? 8. The speed-time graph for a car is shown in the
following figure.
6. The given figure shows the distance-time graph
of three objects A, B and C. Study the graph and
answer the following questions:

(a) Find out how far the car travels in the first 4
seconds. Shade the area on the graph that
represents the distance travelled by the car
during the period.

(b) Which part of the graph represents uniform


motion of the car?

9. State which of the following situations are possible


(a) Which of the three is travelling the fastest? and give an example for each of these:

(b) Are all three ever at the same point on the (a) an object with a constant acceleration but with
road? zero velocity.

(c) How far has C travelled when B passes A? (b) an object moving in a certain direction with
an acceleration in the perpendicular direction.
(d) How far has B travelled by the tim e it
passes C? 10. An artificial satellite is moving in a circular orbit of
radius 42250 km. Calculate its speed if it takes 24
hours to revolve around the earth?


CORPORATE OFFICE : Parus Lok Complex, Boring Road Crossing, Patna - 1


[ 40 ] PHYSICS | STD.IX

C ORNER
EXERCISE-1
(BASED ON JEE MAIN/ NEET PATTERN)
SCAQ Single Correct Answer Questions 6. The velocity time graphs of a body is as shown.
The displacement covered by it is
DIRECTIONS: Each question has 4 choices (A), (B), (C) V
and (D) out of which only one is correct.
1. A body projected vertically upwards crosses a point
twice in its journey at a height h just after t1 and t2 B E
10 m/s
seconds. Maximum height reached by the body is

 t1  t 2 
2
g
(A)  t1  t 2  (B) g  
2
O 10 20 30 50
 t
 4 
A C D F
4

 t1  t 2 
2 (A) 300m (B) 400m (C) 250m (D) 200m
g
(C) 2g  
 (D)  t1t 2  7. A particle starts moving from rest under uniform
 4  4
acceleration. It travels a distance x in the first two
2. A car moving with a speed of 50km/hr can be stopped seconds and a distance y in the next two seconds. If
by brakes after atleast 6m. If the same car is moving y = nx, then n=..........
at a speed of 100km/hr, the minimum stopping
distance is (A) 1 (B) 2 (C) 3 (D) 4
(A) 12m (B) 18m (C) 24m (D) 6m 8. Initial velocity of a particle moving along a straight
3. A bullet on entering a target with an initial velocity u, line is 10 m/s and its retardation is 2 m/s2. Distance
covered by the particle in the fifth second of its motion
u
loses of its velocity after penetrating a distance x is
n
into the target, how much further will it penetrate? (A) 1m (B) 19m (C) 50m (D) 75m

x  n  1 x  n  1 9. Tripling the speed of a motor car multiples the


2 2

(A) S  (B) S  distance needed for stopping it by ___________


 2n  1  2n  1 times.
x  n  1 x  n  1
2

S (A) 3 (B) 6
S
(C)
 2n  1 (D)
 2n  1 (C) 9 (D) Some other number
4. Velocity time curve for a body projected vertically
upwards is 10. A motorist travels from A to B at a speed of 40 kmph
and returns back at a speed of 60 kmph. His average
(A) Parabola (B) Ellipse
speed will be
(C) Hyperbola (D) Straight Line
5. A starts from rest and moves with acceleration a1. (A) 40 kmph (B) 48 kmph
Two seconds later, B starts from rest and moves (C) 50 kmph (D) 60 kmph
with an acceleration a2. If the displacement of A in
the 5th second is the same as that of B in the same 11. A car moving at a speed of 20 m/s undergoes uniform
interval, the ratio of a1 to a2 is retardation of 5 m/s2. It stops in a time of
(A) 9:5 (B) 5:9 (C) 1:1 (D) 1:3 (A) 100 s (B) 4 s (C) 3 s (D) 5 s

CORPORATE OFFICE : Parus Lok Complex, Boring Road Crossing, Patna - 1


CBSE | MOTION [ 41 ]

12. A car is moving along a straight road with a uniform If the total time taken is 4s, find the total distance
acceleration. It passes through two points P and Q travelled by the car.
separated by a certain distance with velocity of 30
kmph and 40 kmph respectively. Velocity of the car, (A) 16 m (B) 8 m (C) 24 m (D) 32 m
exactly midway between P and Q, is 16. A particle starting from rest from O accelerates
(A) 33.3 kmph (B) 20 kmph uniformly along a straight line and in 2 s crosses
two points A and B, which are 5 m apart. If velocity
(C) 25 kmph (D) 35.35 kmph at B is 2 m/s more than the velocity at A, find the
13. Speeds of two identical cars are u and 4u at a specific distance OA.
instant. The ratio of the respective distances in which
the two cars are stopped from that instant is 9 9 9
(A) 8 m (B) m (C) m (D) m
(A) 1:1 (B) 1:4 (C) 1:8 (D) 1:16 8 2 4
14. The distances travelled by a body, starting from rest 17. Two cars are moving on a straight road with the one
and travelling with uniform acceleration in successive car 200m away from the other moving at the same
intervals of time of equal duration, will be in the ratio speed. At a certain moment, brakes are applied to
of the rear car, which decelerates uniformly, travelling
(A) 1:2:3 (B) 1:2:4 (C) 1:3:5 (D) 1:4:9 20 m before coming to rest. Find the distance
between the cars at the moment when the rear car
15. A car accelerates from rest at a constant rate for comes to rest.
some time and attains a velocity of 8 m/s. Afterwards,
it decelerates at a constant rate and comes to rest. (A) 200 m (B) 240 m (C) 220 m (D) 180 m

EXERCISE-2
(BASED ON JEE ADVANCED PATTERN)
MCAQ Multi Correct Answer Questions 3. Choose the incorrect statement/s:
(A) The magnitude of the velocity of a particle must
DIRECTIONS: Each question has 4 choices (A), (B), (C) and be is equal to its speed.
(D) out of which one or more than one is correct. (B) The magnitude of average velocity in an interval
1. For a particle moving along a straight line. Choose is equal to its average speed in that interval.
the correct statement(s). (C) It is possible to have a situation in which the
speed of a particle is always zero but the
(A) Magnitude of the displacement may be equal average speed is not zero.
to the distance covered.
(D) It is possible to have a situation in which the
(B) Magnitude of the displacement always equal speed of the particle is never zero but the
to the distance covered. average speed in an interval is zero.
4. If an object covers unequal distances in equal
(C) Magnitude of the displacement will never be intervals of time. Then object is said to move with
equal to distance.
(A) variable speed (B) constant speed
(D) Magnitude of the displacement may be zero (C) non-uniform speed (D) uniform speed
where as the distance will not be zero.
5. Choose the correct statements from the following.
2. Choose the correct statement/s (A) A body having a constant speed cannot have a
(A) The SI unit of displacement is metre. varying velocity.
(B) A body having a constant speed can have a
(B) Displacement is a scalar quantity. varying velocity.
(C) For a given motion, the distance is equal to or (C) A body having constant speed can have an
greater than the magnitude of displacement. acceleration.
(D) If velocity and acceleration are in same
(D) For a body in straight line motion, the magnitude direction, then distance is equal to
of distance and displacement are same. displacement.

CORPORATE OFFICE : Parus Lok Complex, Boring Road Crossing, Patna - 1


[ 42 ] PHYSICS | STD.IX

6. Mark the correct statements for a particle going in a 10. The velocity-time plot for a particle moving on a
straight line. straight line is shown in figure.
(A) If the velocity and acceleration have opposite v (m/s)
sign, the object is slowing down.
10 A
(B) If the position and velocity have opposite sign, B C
the particle is moving towards the origin.
0
10 20 30 t (s)
(C) If the velocity is zero at an instant, the
acceleration should also be zero at that instant. 10 
D
(D) If the velocity is zero for a time interval, the 20 
acceleration is zero at any instant within the
time interval. Choose the right option.
7. A car accelerates from rest at a constant rate of 2 (A) The particle has a constant acceleration.
ms–2 for some time. Then it retards at a constant (B) The particle has never turned around.
rate of 4 ms–2 and comes to rest. It remains in motion
for 6 s. (C) The particle has zero displacement.
(D) The average speed in the interval 0 to 10s is
(A) Its maximum speed is 8 ms–1. the same as the average speed in the interval
10s to 20s.
(B) Its maximum speed is 6 ms–2.
11. Choose the correct statements
(C) It travelled a total distance of 24 m.
(A) Graph is a pictorial representation of two
(D) It travelled a total distance of 18 m. physical variables recorded by an experimenter.
8. Which is/are correct ? (B) In a graph one variable is independent where
as other variable is dependent.
(A) If velocity of a body changes, it must have some
acceleration. (C) In a graph the independent variable is always
plotted on X- axis.
(B) If speed of a body changes, it must have some
acceleration. (D) In a graph the dependent variable is always
plotted on Y- axis.
(C) If body has acceleration, its speed must
change. STQ Statement Type Questions
(D) If body has acceleration, its speed may change.
DIRECTIONS: Each question in section has four choices (A),
9. A and B start their journey towards their homes ‘P’ (B), (C) and (D) out of which only one is correct. Mark
and ‘Q’ from different points. Which of the following your choices as follows:
statements is correctly represented by the graph
given below? (A) Statement-I is True, Statement-II is True; Statement-
II is a correct explanation for Statement-I
(B) Statement-I is True, Statement-II is True; Statement-
II is NOT a correct explanation for Statement-I
(C) Statement-I is True, Statement-II is False
(D) Statement-I is False, Statement-II is True.
12. Statement I : No distinction (as average speed
(A) A starts his journey earlier. and average velocity) is necessary
when we consider instantaneous
(B) B is faster than A. speed and magnitude of velocity.
(C) A and B meet during their journey. Statement II : The instantaneous speed is always
equal to the magnitude of
(D) A and B reach their homes at the same time.
instantaneous velocity.

CORPORATE OFFICE : Parus Lok Complex, Boring Road Crossing, Patna - 1


CBSE | MOTION [ 43 ]

13. Statement I : The units of distance and


displacement are same. CTQ Comprehension Type Questions

Statement II : Both distance and displacement are DIRECTIONS: Each set in this section contains a paragraph
the measurements of length. or comprehension followed by questions. Each question has
four choices (A), (B), (C) and (D), out of which one or
14. Statement I : A body can have non-zero distance
more answers are correct.
and zero displacement.
COMPREHENSION-1 (For Q.no 22-24)
StatementII : A body can have non-zero
displacement and zero distance. A man walks on a straight road from his home to a market
2.5 km away with a speed of 5 km h –1.
15. Statement I : If velocity is constant, speed will also
Finding the market closed, he instantly turns and walks
be constant.
back home with a speed of 7.5 km h–1.
Statement II : If speed is constant, velocity may 22. What is the magnitude of average velocity?
or may not be constant.
(A) 0 (B) 4 km/h (C) 3 km/h (D) 2 km/h
16. Statement I : Speed can be positive or negative.
23. What is the magnitude of average speed of the man
Statement II : Speed is a scalar quantity. over the interval of time 0 to 30 min ?

17. Statement I : The magnitude of velocity can be (A) 2 km/h (B) 3 km/h (C) 4 km/h (D) 5 km/h
positive or negative. 24. What is the magnitude of average speed of the man
over the interval of time 0 to 50 min ?
Statement II : Velocity is a vector quantity.
(A) 4 km/h (B) 5 km/h (C) 6 km/h (D) 7 km/h
18. Statement I : The equation of motion can be
applied only if acceleration is along COMPREHENSION -2 (For Q.no 25-27)
or opposite to the direction of An object is said to be moving with uniform acceleration
velocity and is constant. if its velocity changes by equal amounts in equal intervals
Statement II : If the acceleration of a body is of time.
constant, then it’s motion is known Formulae for uniformly accelerated motion along a straight
as uniformly accelerated motion. line are :
19. Statement I : Equation of motion is represented
1
by v2  u2  2as . (i) v = u + at (ii) s = ut + at 2
2
Statement II : Equations of motion is applied when a
a body is moving along a straight (iii) v2 – u2 =2as (iv) Snth  u + (2n  1)
2
line with uniform acceleration.
(Where ‘u’ is initial velocity, ‘v’ is final velocity, ‘a’ is
20. Statement I : A straight line graph shows that two acceleration (uniform), ‘t’ is total time, ‘s’ is the
variable quantities are directly pro- displacement. Snth is the distance travelled in nth second
portional to one another. of uniformly accelerated motion.)
Statement II : A graph helps to determine the na- 25. A train starts from rest with an acceleration of
ture of proportionality relation be- 2 ms–2. A man who is 9m behind the train runs with
tween the variable quantities. uniform speed and just manages to get into the train
in 10sec. The speed of the man is
21. Statement I : The velocity-time graph for a body
moving with uniform acceleration is (A) 9 ms–1 (B) 8.7 ms–1
a straight line passing through (C) 4.5 ms–1 (D) 10.9 ms–1
origin.
26. A body moving with uniform acceleration covers 60
Statement II : When the size of each step in a step m in the first 5 seconds and 100 m in the next 5
wise increasing speed-time graph seconds. The initial velocity of the body is
becomes very small, the speed-
(A) 15 ms–1 (B) 8 ms–1
time graph becomes a straight line.
(C) 5 ms–1 (D) 2.5 ms–1

CORPORATE OFFICE : Parus Lok Complex, Boring Road Crossing, Patna - 1


[ 44 ] PHYSICS | STD.IX

27. A person travelling at 54 km/h applies the brake


giving a deceleration of 5.0m/s2 to his scooter. How MMTQ Matrix Match Type Questions
far will it travel before stopping?
DIRECTIONS: Following questions has four statements
(A) 16 m (B) 20 m (C) 22.5 m (D) 18 m (a, b,c and d) given in List -1 and four or more statements
COMPREHENSION -3 (For Q.no 28-30) (p, q, r, s, t...) in List-2. Any given statement in List-1
can have correct matching with one or more statement(s)
A particle is moving along a straight line with constant
given in List-2. Match the entries in List-1 with entries
acceleration. At a point A on the line its velocity is equal to
4 m/s and at a point B its velocity becomes 6 m/s. The in List-2.
distance between these two points is 2 m. For example, if correct matches are a-p, a-s, b-q, b-r, c-p,
28. Find the acceleration of the particle. c-q and d-s, then the correctly bubbled 4×4 matrix should
be as follows.
(A) 2 m/s2 (B) 5 m/s2
(C) 7 m/s2 (D) 9 m/s2
29. Find the time taken by the particle to go from A to B.
(A) 0.2 s (B) 0.4 s
(C) 0.6 s (D) 0.8 s
30. If at time t = 0, particle is at point A, find the distance
moved by the particle in the 5th second. 34. A wall clock has its minute hand 5.0 cm long. Match
the average velocity of the tip of the minute hand
(A) 26.5 m (B) 27.5 m with the respective timings.
(C) 28.5 m (D) 29.5 m List-1 List-2
COMPREHENSION-4 (For Q.no 31-33)
1
The graph given here shows the positions of the body at (a) 3.00 pm to 3:15 pm (p) cm / s
different times. 45

7 D 2
(b) 3.00 pm to 3:30 pm (q) cm/s
6 540
5
Displacement (cm)

4
1
(c) 3.00 pm to 3:45 pm (r) cm / s
B C 180
3
2
2
1 (d) 3.00 pm to 4:00 pm (s) cm / s
A 180
2 3 4 5 6 7 8 9 (t) Zero
Time (s)

31. Calculate the speed of the body as it moves from A 35. A body covers 12m in 2nd second and 20m in 4th
to B. second. Then
(A) 0.2 cm/s (B) 0.4 cm/s List-1 List-2
(C) 0.6 cm/s (D) 0.8 cm/s (a) the uniform acceleration (p) 24
32. Calculate the speed of the body as it moves from B 2
of the body in m / s is
to C.
(b) the initial velocity of the (q) 16
(A) 0 cm/s (B) 1 cm/s body in m/s is
(C) 2 cm/s (D) 3 cm/s
(c) the distance covered by (r) 6
33. Calculate the speed of the body as it moves from C the body in 5th sec
to D. in metres is
(A) 0 cm/s (B) 1 cm/s (d) the distance covered by (s) 4
(C) 2 cm/s (D) 3 cm/s the body in 3rd second in
metres is

CORPORATE OFFICE : Parus Lok Complex, Boring Road Crossing, Patna - 1


CBSE | MOTION [ 45 ]

36. The displacement – time graph is shown below


ITQ Integer Type Questions
A
DIRECTIONS: The answer to each of the questions is a
E single digit non negative integer, ranging from 0 to 9.
D 38. A man leaves his house for a cycle ride. He comes
back to his house after half-an-hour covering a
B C distance of one km, then the magnitude of average
velocity for the ride is ___________ km/h.
39. A car travels equal distances in the same direction
with velocities 60 kmh–1, 20 kmh–1 and 10 kmh–1
respectively. The magnitude of average velocity of
From the graph match the following given below the car over the whole journey of motion
is________m/s.
List-1 List-2
40. An object undergoes an acceleration of 4 m/s 2
–1
(a) Velocity along AB (p) 1.25 ms starting from rest. The distance travelled in metre in
2 second is ___________.
(b) Velocity along BC (q) Zero
41. A particle moving in a straight line covers half the
(c) Velocity along DE (r) –7.5 ms–1
distance with speed of 3m/s. The other half of the
(d) Average velocity along CE (s) 2.14 ms–1 distance is covered in two equal time intervals with
speed of 4.5 m/s and 7.5 m/s respectively. The
37. Match column-I with column -II using the following average speed of the particle is ___________ m/s.
graph.
42. A car travelling at 60 km/h overtakes another car
travelling at 42 km/h. Assuming each car to be
Displacement in (m)

14 5.0 m long, find the time taken during the overtake


12
10 in seconds.
8 43. Two stones are released from rest, one from a height
6 of 20 m and another from some unknown height
4 above the ground. The stone released from 20 m
height hits the ground 1 s after the other strikes the
2
ground. The height from which the second stone is
released is _________ m. ( g = 10 ms–2 )
1 2 3 4 5 6 7 8 9
Time in (seconds) 44. The ratio of average speed of a scooter moving at
List-1 List-2 n
10 m/s and a car moving at 27 km/h is . Find
(a) Velocity between 0 - 4s (p) 2 m/s n 1
the value of n.
(b) Velocity between 4s - 6s (q) zero
45. If a sports car can go from rest to 27 ms–1 in 9.0
(c) Average velocity (r) 1.33 m/s second. The magnitude of its average acceleration
between 0 - 6s is _____________ m/s2.
(d) Average velocity (s) 1.55 m/s
46. The brakes applied to a car produce an acceleration
between 0 - 9s of 6 ms–2 in the opposite direction to the motion. If
the car takes 1s to stop after the application of
brakes, then the distance it travels during this time
is __________ m.
47. A 100 m long train crosses a bridge of length 200 m
in 50 seconds with constant velocity. The velocity of
train is _______________ m/s.


CORPORATE OFFICE : Parus Lok Complex, Boring Road Crossing, Patna - 1


[ 46 ] PHYSICS | STD.IX

KEY & ANSWERS


Daily Practice Problem-1 (b) The velocity of the lift at any point of time is
given by the slope of its displacement-time
1. 10 cm graph at that point. The graph shows that slope
of the curve increases gradually from A to B,
2. (a) 110 m reaches a maximum at B(t = 5 s) and then
decreases to become zero at C (t = 7 s).
(b) 50 m towards 53° east of north
v
 1
  with the
–1
3. 6.02 km, along the direction tan
 12  vmax
positive x–axis

4. 5. 37.5 kmph 0 time(s)


3r 5s 7s
2v v
1 2
6. 60 kmph 7. 5. (A)
v v
1 2 6. (B)
2a
8. 9. 60.71 kmph 7. 3m
t
8. 2:1
10. 32km/hr
9. distance = 100 m, Average velocity = 0
Daily Practice Problem-2 10. 10m/s
1. 4m 2. 185m Daily Practice Problem-4
1. (B) 2. (D)
1
3. a= ms–2 ; S = 3.75 km 3. (A) 4. (D)
12
5. (A) 6. (B)
4. 12m 5. 7.5ms–1
7. 20 3 N 8. (B)
  
6. v t 7. 10 ms –2

 9. 231.2 kmph, 115.6 kmph.


8. 10 sec. 9. (D) 10. (C)
10. (A)
Daily Practice Problem-5
1. (B) 2. (D) 3. (D)
Daily Practice Problem-3
4. (A) 5. (D) 6. (D)
1. No, in graph time and distance decreases, which is
not possible. 7. (B) 8. (B) 9. (C)
2. (A) 10. (D) 11. (A) 12. (A)
3. (D) 13. (B) 14. (B) 15. (A)
4. (a) At A : The lift is on the ground floor and is
stationary, i.e., it is at rest. Daily Practice Problem-6
At B : The lift is under accelerated motion from 1. (B) 2. (B) 3. (D)
A to B. At B, its velocity is the maximum: the 4. (A) 5. (A) 6. (A)
slope of the curve at point B is the highest.
7. (B) 8. (D)
At C: The slope of displacement-time graph
is zero. So, the lift is at a halt at some higher 9. 5 m/s towards 37° south of east 10. 20 s
floor.

CORPORATE OFFICE : Parus Lok Complex, Boring Road Crossing, Patna - 1


CBSE | MOTION [ 47 ]

NCERT SECTION PRACTICE


CORNER
INTEXT
1. Conceptual
2. 14.1 m
3. (a) Not true
Displacement can become zero when the initial and
final position of the object is the same. (ii) For non-uniform motion
(b) Not true
Displacement is the shortest measurable distance
between the initial and final positions of an object. Its
magnitude cannot be greater than the distance
travelled by an object. However, sometimes, it may
be equal to the distance travelled by the object.
Speed Velocity
Speed is the distance travelled by an Velocity is the displacement of an
object in a given interval of time. It object in a given interval of time. It 13. A straight line parallel to the x-axis in a distance –
does not have any direction. has a unique direction.
time graph indicates that with a change in time, there
Speed is given by the relation: Velocity is given by the relation: is no change in the position of the object. Thus, the
Distance travelled Displacement object is at rest.
Speed= Velocity =
4. Time taken Time interval 14. A straight line parallel to the time axis in a speed”time
The speed of an object can never be The velocity of an object can be graph indicates that with a change in time, there is
negative. At the most, it can become negative, positive, or equal to zero. no change in the speed of the object. This indicates
zero. This is because distance This is because displacement can
travelled can never be negative. take any of these three values. the uniform motion of the object.
15. Displacement
Total distance covered 16. (a) 12 m/s (b) 720 m
5. Average speed  ; 17. 625 m 18. 0.06 m/s.
Total time taken 19. 200 m. 20. 1.25 m and 0.5 s.
Displacement
Average velocity  NCERT EXERCISE
Total time taken 1. (a) distance = 2200 m; displacement = 200 m.
If the total distance covered by an object is the same 2. (a) average speed = average velocity = 2.00 m s–1
as its displacement, then its average speed would (b) average speed = 1.90 m s–1;
be equal to its average velocity. average velocity = 0.952 m s–1
6. The odometer of an automobile measures the
3. average speed = 26.67 km h–1
distance covered by an automobile.
4. distance travelled = 96 m
7. An object having uniform motion has a straight line
path. 5.
8. 9 × 1010 m.
9. (i) A body is said to have uniform acceleration if it
travels in a straight path in such a way that its velocity
changes at a uniform rate, i.e., the velocity of a body
increases or decreases by equal amounts in an equal
interval of time.
(ii) A body is said to have non-uniform acceleration if
it travels in a straight path in such a way that its
velocity changes at a non-uniform rate, i.e., the
velocity of a body increases or decreases in unequal
amounts in an equal interval of time.
6. (a) Object B (b) No
10. – 1.112 m/s
(c) 7.42 km (d) 5.143 km
11. 0.0185 m/s2.
–1
12. (i) For uniform motion 7. velocity = 20 m s ; time = 2 s

CORPORATE OFFICE : Parus Lok Complex, Boring Road Crossing, Patna - 1


[ 48 ] PHYSICS | STD.IX

8. (a) 12 m acceleration due to gravity, which is equal to 9.8 m/


(b) Between time 6 s to 10 s represents uniform s 2.
motion of the car. (b) Possible
9. (a) Possible W hen a car is moving in a circular track, its
When a ball is thrown up at maximum height, it has acceleration is perpendicular to its direction.
zero velocity, although it will have constant 10. Speed = 3.07 kms–1

COMPETITIVE CORNER
EXERCISE-1
1. (C) 2. (C) 3. (A) 4. (D) 5. (B) 6. (C)

7. (C) 8. (A) 9. (C) 10. (B) 11. (B) 12. (D)

13. (D) 14. (C) 15. (A) 16. (B) 17. (C)

EXERCISE-2
1. (A, D) 2. (A, C) 3. (B, C, D) 4. (A, C) 5. (B, C, D) 6. (A, B, D)

7. (A, C) 8. (A, B, D) 9. (A, B, C) 10. (A, D) 11. (A, B, C, D) 12. (A)

13. (A) 14. (C) 15. (B) 16. (D) 17. (A) 18. (B)

19. (B) 20. (A) 21. (B) 22. (A) 23. (D) 24. (C)

25. (D) 26. (B) 27. (C) 28. (B) 29. (B) 30. (A)

31. (C) 32. (A) 33. (C)

34. (a)  (s) 35. (a)  (s) 36. (a)  (r) 37. (a)  (p)

(b)  (r) (b)  (r) (b)  (q) (b)  (q)

(c)  (q) (c)  (p) (c)  (p) (c)  (r)

(d)  (t) (d)  (q) (d)  (s) (d)  (s)

38. 0 39. 5 40. 8 41. 4 42. 2 43. 5

44. 4 45. 3 46. 3 47. 6



CORPORATE OFFICE : Parus Lok Complex, Boring Road Crossing, Patna - 1

You might also like